SlideShare une entreprise Scribd logo
1  sur  19
Télécharger pour lire hors ligne
Matemática

Pregunta N.º 20                                   Ejecución del plan
                                                  I.
Sea p(x) el polinomio de grado n, donde n es
el menor posible y cuya gráfica se representa a
continuación.




                                                     p(x)=k(x – 1)2a(x – 2)2b – 1;

                                                     a, b ∈ Z+
Encuentre el residuo al efectuar la división de      Como el grado de p(x) es el menor posible,
p(x) con q(x)=x – 3.                                 entonces
                                                           a=1 y
A) – 6                                                     b=1
B) – 4                                               Luego, tenemos
C) – 1
                                                           p(x)=k(x – 1)2(x – 2)
D) 1
                                                     De la gráfica
E) 4
                                                           p(0)=2
Solución                                                   p(0)=k(–1)2(–2)

Tema                                                       p(0)=2
                                                           → k=–1
Gráfica de funciones polinomiales
                                                     Luego
                                                           p(x)=–(x – 1)2(x – 2)
Referencias
                                                  II. Aplicando el teorema del resto tenemos
Para la solución del problema se necesita
                                                            p( x)
conocer:
                                                            x−3
• Gráfica de una función polinomial.
                                                           → R(x)=p(3)
• Teorema del resto.
                                                           p(3)=–(2)2(1)

Análisis y procedimiento                             ∴     p(3)=– 4

Plan de resolución
                                                  Respuesta
I. A partir de la gráfica, hallar la regla de
   correspondencia de p(x).                       El residuo de dividir p(x) entre x – 3 es – 4.

II. Aplicar el teorema del resto.                                                      Alternativa B

                                                                                                       15
Matemática

Pregunta N.º 21
En la figura mostrada ABCD es un cuadrado de
lado 2R, además BC es diámetro de la semicircun-
ferencia de centro O y radio de longitud R. Si T es
un punto de tangencia entonces m TOA es




                                                      Como        ABCD es un cuadrado
                                                           →     BC=CD=2(BO)=2(OC)=2R
                                                      Trazamos OD → OD: Bisectriz del        CDT

                                                                                53º
                                                      Luego,      OCD (not.         ):
                                                                                 2
                                                                        53º
                                                           m CDO=              y
A) 7,5                                                                   2
B) 8                                                                   53º
                                                           m ODT=
C) 10                                                                   2
D) 10,5                                               En        TOCD: inscriptible
E) 12,5
                                                           → m BOT=m CDT
                                                           m BOT=53º
Solución
                                                                      53º
Tema                                                    OBA (not          )
                                                                       2
Circunferencia                                                                53º
                                                           → m BAO=
                                                                               2
Referencias
                                                      En    OBA
En la pregunta nos piden la medida de un ángulo;
                                                                      53º
entonces, debemos ubicarlo en una figura donde             53º+x+         =90º
                                                                       2
se puede obtener dicha medida; por ejemplo,
                                                                21º
un triángulo; además, como se observa una                  x=
                                                                 2
semicircunferencia debemos aplicar los teoremas
                                                           → x=10,5º
que se cumplen en la circunferencia.

                                                      Respuesta
Análisis y procedimiento
                                                      La medida del ángulo TOA es 10,5º.
En el gráfico,
nos piden x.                                                                             Alternativa D

                                                                                                         16
Matemática

Pregunta N.º 22                                        QR // DB
ABC es un triángulo rectángulo. Exteriormente a                                           BD
los catetos se construyen los triángulos equiláteros      → m RQC=150º y RQ=
                                                                                           2
ABD y BEC. P, Q y R son puntos medios de BE,
BC y DC respectivamente. Si el área de la región       PQ // EC
triangular ABC es 32 cm2, entonces el área de la          → m PQC=120º y
región triangular PQR (en cm2) es
                                                                  EC
                                                          PQ=
                                                                   2
A) 4             B) 6              C) 8
D) 12                              E) 16               Luego
                                                          m PQR=90º
Solución                                               En el gráfico,
Tema                                                     PQR ~      ABC (caso LAL de razón 1/2)
Área de regiones triangulares
                                                       Por áreas de regiones semejantes
                                                                                    2
Referencias                                                A PQR    ⎛ razón de ⎞
                                                                   =⎜           ⎟
Para relacionar las áreas de dos regiones trian-           A ABC    ⎝ semejanza ⎠
gulares, se busca la relación entre los elementos
                                                       Reemplazamos
de ambos triángulos (lados, alturas, medida de
                                                                           2
ángulos, etc.).                                            A PQR    ⎛1⎞
                                                                   =⎜ ⎟
                                                             32     ⎝ 2⎠
Análisis y procedimiento
                                                          → APQR=8


                                                       Respuesta
                                                       El área de la región triangular PQR (en cm2) es 8.

                                                                                          Alternativa C



                                                       Pregunta N.º 23
                                                       Indique la secuencia correcta después de determi-
                                                       nar si la proposición es verdadera (V) o falsa (F).
                                                       I. Si dos planos son perpendiculares a dos rectas
Piden   APQR: área de la región triangular PQR.
                                                           diferentes que se intersectan, entonces dichos
Dato    A ABC: área de la región triangular ABC.           planos también se intersectan.
        (A ABC=32)                                     II. El lugar geométrico que determinan los pies de
                                                           los segmentos oblicuos de longitudes iguales
Por ser P, Q y R puntos medios, se determinan              trazadas desde un punto exterior a un plano
bases medias en los triángulos BEC y DBC.                  es una circunferencia.

                                                                                                             17
Matemática

III. Toda recta es perpendicular a un plano, si es    II.
     ortogonal a dos rectas diferentes no paralelas
     contenidas en dicho plano.

A)   VVF
B)   VFV
C)   FFV
D)   VVV
E)   FFF


Solución                                                    • Como el punto Q es exterior al plano, traza-
Tema                                                          mos QQ' de modo que Q' sea la proyección
Geometría del espacio. Rectas y planos                        ortogonal de Q sobre el plano W.
                                                            • En el gráfico, los triángulos rectángulos
Referencias                                                   AQ'Q; BQ'Q y DQ'Q son congruentes
En este tipo de preguntas debemos hacer una                   entre sí.
comparación entre los conceptos teóricos y los              • Luego, m=n=p=…
casos posibles que plantean las proposiciones. De           • Además, el punto Q' equidista de A, B,
esta manera, determinamos la veracidad o falsedad             C, D, …
de la proposición dada.                                       Por lo tanto, el lugar geométrico que deter-
                                                              minan A, B, C y D es una circunferencia de
Análisis y procedimiento                                      centro Q'.
Esta pregunta consta de tres proposiciones.                   Entonces, la proposición es verdadera.
I. En el espacio, solo se admiten dos posiciones
                                                      III. En el gráfico, para que una recta sea perpendicular
   relativas entre dos planos: son paralelos o son
                                                           a un plano, debe ser perpendicular a dos rectas
   secantes.
                                                           no paralelas contenidas en dicho plano.




                                                            Entonces, la proposición es verdadera.
     • En la fig.1, los planos son paralelos si son
       perpendiculares a una misma recta.             Respuesta
     • En la fig. 2, los planos son secantes si son   La secuencia correcta después de analizar las
       perpendiculares a dos rectas que se interse-   proposiciones es VVV.
       can (proposición de la pregunta).
       Entonces, la proposición es verdadera.                                               Alternativa D

                                                                                                                 18
Matemática

Pregunta N.º 24                                     Análisis y procedimiento

En la figura mostrada, ABCD es un trapecio          Piden
rectángulo tal que CD=BC=2AB=2a. Si PQ es           Volumen de la pirámide Q-BCP:
perpendicular al plano del trapecio tal que PQ=a
                                                               1
y los volúmenes de las pirámides Q-ABP y Q-CDP          Vx =     [ A BCP ][PQ]                        (I)
                                                               3
son iguales, calcule el volumen de la pirámide
Q-BCP.




                                                    Del gráfico tenemos PQ=a                (II)
                                                    Como los volúmenes de las pirámides Q-ABP y
                                                    Q-PCD son iguales, al tener la misma altura, las
A) 1 a 3                          B) 3 a 3
                                                    áreas de sus bases son también iguales.
   2                                 8
                                                    Entonces,
                                                       AABP=ACPD=4A.
     4 3
C)     a
     5                                              En el plano de la base

     7 3                               5 3
D)     a                          E)     a
     8                                 9



Solución
Tema
Geometría del espacio. Pirámide


Referencias
En preguntas donde piden el cálculo o la relación
de volúmenes, conviene hacer un análisis de las     Del dato de áreas iguales
                                                       → AP=2(PD)
longitudes de las alturas o de las relaciones de
                                                    Por relación de áreas, el área de la región trapecial:
las bases. Generalmente, para el cálculo del área
de la base se emplean capítulos anteriores de                  ⎛ a + 2a ⎞            a2
                                                        18 A = ⎜        ⎟ (2a) → A =
geometría plana.                                               ⎝ 2 ⎠                 6

                                                                                                             19
Matemática

Luego,
                   5a 2
   ABCP=10A=                                    (III)
                    3
Reemplazamos (II) y (III) en (I)

              1 ⎛ 5a 3 ⎞       5a 3
   → Vx=        ⎜      ⎟ (a) =
              3⎝ 3 ⎠            9
                                                        y para poder aprovechar el ángulo de inclinación
Respuesta                                               es preciso asociarlo con el teorema de las tres
                                           5a   3       perpendiculares.
El volumen de la pirámide Q-BCP es
                                            9
                                                        Análisis y procedimiento
                                      Alternativa E
                                                        Graficamos el prisma según las condiciones
                                                        planteadas.

Pregunta N.º 25                                          D'                        C'
                                                                                                            M
La altura de un prisma recto mide 1 u, su base es                                       S'                           B'
una región limitada por un rombo cuyo lado mide         1u                     N
                                                                                        A'
2 u y su ángulo agudo mide 30º. Por un lado de                          2u         C              h
                                                                                             1u
la base se traza un plano que interseca al prisma                 30º
                                                                                                       2u
                                                         D
y está inclinado un ángulo de 60º con respecto                          2u                   S
                                                                                                      60º      30º
                                                                                                                     B
de la base, luego el área de la sección (en u2) que                                                   H
                                                                                         A                     3u
resulta en el prisma es:                                                                                  2u

                       5                   4
A) 2 3            B)                  C)                donde ABCD es un rombo de lado 2 u y la
                        3                   3
                                                        m ABC=30º.
   3                                     2
D)                                    E)
    3                                     3             Si trazamos

                                                              CH ⊥ AB ... 1.a ⊥

Solución                                                      SS' ⊥ CH ... 2.a ⊥
Tema                                                          → S'H ⊥ AB ... 3.a ⊥
Prisma
                                                        Sea S'H=h.

Referencias                                             Como la altura del prisma es 1 u
                                                          → S'S=1 u
Al trazar planos secantes a un sólido, este determina
                                                        Luego, en el         S'SH:
secciones planas, que varían de acuerdo al ángulo
de inclinación y el lugar por donde interseca. Así,           hsen60º=1 u
un plano secante en un prisma puede determinar                          2
                                                              → h=         u
una sección triangular, cuadrangular, ...                                3

                                                                                                                          20
Matemática

Luego, el área de la sección ABMN, que es una           Análisis y procedimiento
región paralelográmica, se calcula multiplicando        Piden r2+3r.
AB y h.
                                                        Las longitudes de los lados del polígono convexo
                                                        de 8 lados están en progresión geométrica de
                               ⎛ 2 ⎞
          ABMN= AB h = ( 2 u ) ⎜
   A           ( )                 u⎟                   razón r.
                               ⎝ 3 ⎠                                              B
                                                                          a            ar
                 4 2                                                 A                         C
    A     ABMN =    u
                  3
                                                               ar7                                 ar2

Respuesta
                                                              H                                      D
                                 4
El área de la sección en u2 es      .
                                  3                                                                ar3
                                                               ar6

                                        Alternativa C                G                         E
                                                                         ar5           ar4
                                                                                  F

Pregunta N.º 26                                         además

Se tiene un polígono convexo de 8 lados circuns-           AB= ´ 1 , BC= ´ 2 , CD= ´ 3 , DE= ´ 4 , EF= ´ 5 ,
crita a una circunferencia, si las longitudes de sus       FG=´6, GH=´7 y HA=´8,
lados están en progresión geométrica de razón r.
                                                        En el octógono circunscrito por el teorema de Pithot
Determine r2+3r.
                                                        general, tenemos:

A) 1             B) 4               C) 10
                                                           ´1+´3+´5+´7=´2+´4+´6+´8
D) 18                               E) 28                  → a+ar2+ar4+ar6=ar+ar3+ar5+ar7
                                                        Factorizamos
Solución                                                   a(1+r2+r4+r6)=ar(1+r2+r4+r6)

Tema                                                       → r=1
Polígonos circunscritos a una circunferencia:
Teorema de Pithot generalizado                          Respuesta
                                                        El valor de r2+3r es 4.
Referencias
En un cuadrilátero circunscrito o circunscriptible,                                          Alternativa B
se cumple el teorema de Pithot, es decir, la suma de
longitudes de lados opuestos son iguales.
En un polígono circunscrito o circunscriptible se
                                                        Pregunta N.º 27
cumple que la suma de longitudes de lugar par
es igual a la suma de longitudes de lugar impar,        Se da un triángulo ABC cuyos lados AB y BC
es considerado para un cuadrilátero, hexágono,          miden 8 m y 6 m respectivamente. Sobre AB
octógono, ..., en polígonos cuyo número de lados        se toma el punto D. Si m BAC=m BCD.
es par.                                                 Entonces AD es:

                                                                                                               21
Matemática

A) 3,5             B) 4                   C) 4,5       Piden AD
D) 5                                      E) 5,5
                                                       Datos:
                                                       AB=8, BC=6
Solución                                               m BAC=m BCD
Tema                                                      ABC: Por teorema de semejanza
Semejanza de triángulos                                tenemos:
                                                          (BC)2=(AB)(BD)                          (I )
                                                       también
Referencias
                                                          BD=8 – AD
Cuando en un triángulo se desea relacionar las         Reemplazamos:
longitudes de lados y segmentos determinados              62=8(8 – AD)
por una ceviana, se puede recurrir a la teoría de         → AD=3,5
semejanza, y más aún si la medida de un ángulo
                                                       Respuesta
es igual al ángulo determinado por dicha ceviana
                                                       Entonces, AD es 3,5.
y un lado; por ejemplo:
                                                                                          Alternativa A
                              B

                                  q
                                           x
                                                       Pregunta N.º 28
                                                       En figura, AB y AC con diámetros, CT es tan-
                                                       gente al arco AB, AB=BC=2r y ET=4. Calcule r.

                                                   C
            q                 M            m
   A
                          b


Teorema:
En el       ABC
m BAC=m MBC=θ
                                                       A) 2 3           B) 2 2           C)   3
   → x2=bm
                                                       D)   6                            E) 3 3
Análisis y procedimiento

                                      B
                                                       Solución
                                                       Tema
                                                       Semejanza de triángulos
                  8D                           6
                                                       Referencias
                                                       En el problema nos piden calcular el radio de la
                                               q       semicircunferencia menor, para ello debemos rela-
            q
        A                                          C   cionar el dato numérico con la variable, utilizando

                                                                                                             22
Matemática

los teoremas que se cumplen en circunferencias       Pregunta N.º 29
tangentes interiores. Luego, para obtener el valor
                                                     En un triángulo ABC se cumple AB=2 m y
del radio debemos establecer una operación que
                                                     AC=32 m. Halle el perímetro del triángulo en
relacione la incógnita con los datos.
                                                     metros, sabiendo que es un número entero y el
                                                     ángulo en A es obtuso.
Análisis y procedimiento

                                     E               A) 65            B) 66             C) 67
                                                     D) 68                              E) 69
          D                    4
                                          4 2
                          T
                  4   a                              Solución
                              2 2
      a                                     a        Tema
          a
  A           r           r   B      2r         C    Clasificación de triángulos:
                                                     Triángulo obstusángulo.
Trazamos BT
     → m BTA=90º                                     Referencias
Por teorema                                          Para realizar el cálculo del perímetro, es necesario
     ET=TA=4                                         conocer BC, el cual, por dato, debe ser entero.
                                                     Como las longitudes de los otros dos lados son
Trazamos AD
                                                     conocidas, podemos restringir a BC mediante el
→ AT es bisectriz del          DAC
                                                     teorema de existencia; pero como la medida de
     m DAT=m TAC=α                                   un ángulo interior es mayor de 90º (obtuso), se
Luego                                                puede realizar la restricción de BC por la naturaleza
                                                     del triángulo.
     m ECD=m DAE=α

En     AEC: Teorema de semejanza                     Análisis y procedimiento
     (EC)2=(8)(4)
                                                     Por dato del problema tenemos
     → EC = 4 2
                                                     AB=2,
  AEC: Teorema base media
                                                     AC=32 y
     → TB = 2 2
                                                     m BAC>90º
                               2
  ATB: (2r)2=42+( 2 2 )
                                                     Piden
     r= 6
                                                        2P       ABC=2+32+BC=34+BC.


Respuesta                                              B

El valor de r es 6.
                                                             2
                                                                                                     C
                                     Alternativa D                               32
                                                                 A

                                                                                                             23
Matemática

En el    ABC: Existencia de triángulos                     27                  54               108
                                                      A)                  B)               C)
                                                           π                    π                π
   32 – 2 < BC < 32+2                      (I)
• Como m BAC>90º
                                                      D) 54                                E) 108
   322+22 < BC2
   32,06 < BC                              (II)
                                                      Solución
• Luego, relacionamos las restricciones (I) y (II).
                                                      Tema
   32,06 < BC < 34                         (III)
                                                      Sólidos geométricos
• 2P     ABC=34+BC

   Como el perímetro es entero, entonces, BC es
                                                      Referencias
   entero.
                                                      Para calcular el volumen de una pirámide se ne-
• Luego, de la expresión (III) obtenemos              cesita conocer el área de su base y la altura de la
   BC=33                                              pirámide, mientras que para calcular el volumen
                                                      del cilindro se requiere conocer el área de su base
∴ 2P     ABC=67
                                                      y su altura. Como el cilindro es circular oblicuo,
                                                      su base es un círculo, mientras que la base de la
Respuesta                                             pirámide es un triángulo equilátero.
El perímetro de la región triangular ABC en metros
es 67.                                                Análisis y procedimiento
                                                      Del gráfico que nos dan como dato podemos no-
                                   Alternativa C      tar que ambos sólidos tienen la misma altura y el
                                                      triángulo de la base de la pirámide está inscrita en la
                                                      circunferencia que limita la base del cilindro.
Pregunta N.º 30                                       Denotemos los vértices de la base de la pirámide
En la figura se tiene una pirámide inscrita en un     como A, B y C, y r el radio del círculo de la base
                                                      del cilindro.
cilindro circular oblicuo. La base de la pirámide
es un triángulo equilátero. El volumen de la
                                                                                    O
              27 3                                                              r
pirámide es        cm3. Calcule el volumen del
                π
cilindro (en cm3).



                                                                                    B
                                                                A
                                                                      r
                                                                          C



                                                      Graficando el triángulo equilátero inscrito en la
                                                      circunferencia tenemos:

                                                                                                                24
Matemática

                                B                             Pregunta N.º 31
                      30º           30º                       En un polígono convexo equiángulo ABCDEF se
                             r            r                   tiene AB=7, CD=6 y DE=8. Calcule BF.
                            O'
                                                                   7
                      r              r                        A)     3                B) 7             C) 5 3
                            120º                                   2

           A                                   C
                            r 2                               D) 7 2                                   E) 7 3



En el     AO'C:
                                                              Solución
                                                              Tema
   AO=r=OC
                                                              Polígonos
   m AOC=120º

   → AC=r 3=AB=BC
                                                              Referencias

Ahora podemos calcular el volumen de la pirá-                 Dentro del grupo de los polígonos tenemos al
mide.                                                         polígono equiángulo, que se caracteriza por que
                                                              sus medidas angulares internas y externas son,
                               2
          1          1 ⎛ (r 3 ) 3 ⎞                           respectivamente, iguales.
   VO-ABC= (Abase)×h= ⎜⎜          ⎟×h
                                  ⎟                           Como se conoce que la suma de las medidas
          3          3⎝      4    ⎠
                                                              angulares de un polígono convexo es 180º(n – 2)
               2                                              y n es el número de lados, entonces, la medida de
   VO-ABC= r          3⋅
                           h=
                                 27 3 cm3
                                                              un ángulo interior será:
                  4                π

De aquí podemos despejar las variables y obte-                          180º ( n − 2 )
                                                                   i=
nemos:                                                                       n

   πr 2 · h=108 cm3                                     (I)
                                                              Análisis y procedimiento
Ahora calculamos el volumen del cilindro
                                                              Según el dato del problema, el polígono equián-
   Vcilindro=A base×h
                                                              gulo es ABCDEF, es decir, tiene seis lados (n=6);
   Vcilindro=πr 2×h
                                                              entonces,
De (I):

   Vcilindro=108 cm3                                                          180º ( 6 − 2 )
                                                                   i( 6 ) =                  = 120º.
                                                                                   6

Respuesta
                                                              Grafiquemos el hexágono con las condiciones del
El volumen del cilindro en cm3 es 108.                        problema:
                                                                   AB=7, CD=6 y
                                              Alternativa E
                                                                   DE=8.

                                                                                                                  25
Matemática

                     B           C                    Pregunta N.º 32
                              120º                    El ángulo de desarrollo de un cono circular recto
             7                            6
                                                      mide 120º. Si la altura del cono mide 4 cm,
          A 120º                     120º D           entonces el radio (en cm) del cono es:
                     x
          60º                          60º
      a      a                       8     8                2
                                                      A)                    B)   2           C)   3
     60º 60º                  120º 60º 60º                 2
   M    a    F                   E      8    N
                                                      D) 2 2                                 E) 2 3

Al prolongar los lados BA, EF y CD, las medidas de
los ángulos externos en A, F, E y D es 60º, además,   Solución
se forman los triángulos AFM y DEN; estos, a la
                                                      Tema
vez, forman el triángulo isósceles MBCN, donde
                                                      Cono circular recto
MB=CN.
Como
                                                      Referencias
   DE=8
   → DN=EN=8.                                         Al desarrollar la superficie lateral de un cono

Así también si                                        circular recto, resulta un sector circular cuyos
                                                      elementos se asocian con los del cono dado.
   AF=a
   → AM=MF=a.
                                                                                              A
Luego                                                                                    g
   a+7=6+8                                                                  V
∴ a=7                                                                            a

Por lo tanto, en el triángulo notable BAF tenemos                                            2pr B
                                                                     g               g
                                                                             h
                          A
                                                                                 r
                 7       120º         7                         B        O               A



  F                       x                   B       En el gráfico α es la medida del ángulo de

Entonces, BF=7 3.                                     desa-rrollo.
                                                      Sea θ su medida en radianes.

Respuesta                                                             πα
                                                           →    θ=
                                                                     180º
La longitud de BF es 7 3.
                                                      Luego, la longitud del arco ABA se asocia con el
                                      Alternativa E   radio de la base del cono.

                                                                                                          26
Matemática

    ´ ABA =2πr                                      Solución
                                                    Tema
    ´ ABA =θ×g
                                                    Sistemas de medición angular
     2πr
∴ θ=
      g
                                                    Referencias

Análisis y procedimiento                            La equivalencia entre los grados sexagesimales y el
                                                    número de radianes de un ángulo es π rad=180º.
Nos dan como dato α=120º y h=4 cm; entonces,
podemos calcular θ y encontrar una relación entre   Análisis y procedimiento
r y g.
                                                    • Nuevo sistema de medición angular (X), donde
                                                      1X denota un grado en el sistema X.
        π (120º ) 2π
   → θ=          =
          180º     3                                • Condiciones:

Luego                                                        αº=(α – 3)X

                                                             π rad=120X
    r 1 ó g=3r
     =                                                 Empleamos el método del factor de conversión:
    g 3

Como nos piden el radio de la base en cm, re-                               ⎛ π rad ⎞ ⎛ 180º ⎞
                                                             αº = (α − 3) X ⎜       ⎟⎜        ⎟
                                                                            ⎝ 120 X ⎠ ⎝ π rad ⎠
currimos al teorema de Pitágoras para relacionar
r, g y h.                                                                        º
                                                                          ⎛3⎞
En el       AVO: g 2=r 2+h2                                  αº = (α − 3) ⎜ ⎟
                                                                          ⎝ 2⎠
Reemplazamos valores:
                                                             2α=3α – 9
   (3r)2=r 2+(4)2
                                                             α=9
∴ r= 2
                                                       Se busca calcular
                                                             (α – 3).
Respuesta
El radio del cono en centímetros es 2.
                                                    Respuesta

                                 Alternativa B      El valor de (α – 3) es 6.

                                                                                         Alternativa B
Pregunta N.º 33
En un nuevo sistema de medición angular, un
ángulo de α grados sexagesimales mide α – 3. Si
                                                    Pregunta N.º 34
                                                                   a 3
un ángulo de π radianes mide 120 en el nuevo        En la figura    = y el área de la región sombreada
                                                                   b 2
sistema, halle α – 3.                               es 5 veces el área del sector circular OPQ.
                                                                            ´ SR
                                                    Determine la relación        .
A) 3               B) 6          C) 9                                       ´ BA
D) 12                            E) 15

                                                                                                          27
Matemática

                                                                                2k      D

                                                                           B
                                                               3k
                                                                           Q                S

                                                     O       q a
                                                                            P

                                                                           A                R

     2                    16            3                                              C
A)                  B)             C)
     3                    27            2
                                              Pero ´ SR = α(5k)
   45                                 10
D)                                 E)
   16                                  3            ´ BA = θ(3k)

Solución
                                                    ´ SR 5 ⎛ α ⎞
Tema                                                    =                                       (I)
                                                    ´ BA 3 ⎜ θ ⎟
                                                           ⎝ ⎠
Longitud de arco y área del sector circular

                                              Condición 2
Referencias
                                              El área sombreada es igual a cinco veces el área
• Longitud de arco (´)                        del sector OPQ.

                                                  1         1           ⎛ α(3k)2 ⎞
             r                                      θ(5k)2 − θ(3k)2 = 5 ⎜
                                                  2         2           ⎜ 2 ⎟    ⎟
                                                                        ⎝        ⎠
         q rad            µ    µ=q×r
                                                  16θk 2 45αk 2
                                                        =
                                                    2      2

• Área de un sector circular (A)                  16 α
                                                    =                                           (II)
                                                  45 θ
             r
                                              Al reemplazar (II) en (I) se obtiene:
         q rad                 A=qr 2
                                  2
                                                  ´ SR 5 ⎛ 16 ⎞
                                                      =
                                                  ´ BA 3 ⎜ 45 ⎟
                                                         ⎝ ⎠

                                                  ´ SR 16
Análisis y procedimiento                              =
                                                  ´ BA 27
Condición 1

     a 3         a = 3k                       Respuesta
      =
     b 2         b = 2k                                     ´ SR    16
                                              La relación        es    .
                                                            ´ BA    27
             ´ SR
Incógnita:
             ´ BA                                                                    Alternativa B

                                                                                                       28
Matemática

Pregunta N.º 35                                             Reemplazamos (II) en (I)

Un punto M=(x; y) dista de un punto C=(2; 5),                                       2
                                                                        ⎛1          ⎞
                                                               (x – 2)2+⎜ ( x − 7 ) ⎟ = 10
    10 unidades. La pendiente de la recta que pasa                      ⎝2          ⎠
por M y A=(7; 5) es 1/2. Determine el punto M
                                                                        1
de mayor abscisa.                                              (x – 2)2+ (x – 7)2=10
                                                                        4

                                                            Reduciendo, tenemos
A) (–1; 4)          B) (–1; 6)               C) (1; 8)
D) (3; 2)                                    E) (5; 4)         x2 – 6x+5=0
                                                               x      –5
Solución                                                       x      –1
Tema                                                           x=5 ∨ x=1
                                                            Piden el punto M de mayor abscisa< enton-
Geometría analítica
                                                            ces, x=5.
Referencias                                                 Reemplazamos en (II)
                                                                     1
• Distancia entre dos puntos                                   y – 5= (5 – 7)
                                                                     2
• Ecuación de una recta
                                                               y=4
                                                            Entonces, M=(5,4).
Análisis y procedimiento
De la condición tenemos                                     Respuesta
•     C(2; 5)                                               El punto M de mayor abscisa es (5,4).

                 10                                                                            Alternativa E
                            M (x ; y )

     Por distancia entre dos puntos se cumple que           Pregunta N.º 36
                           2            2                   En el círculo trigonométrico de la figura, se tiene
             10 = ( x − 2 ) + ( y − 5 )
                                                            CM = DM . Entonces el área de la región triangular
     Elevando al cuadrado, tenemos
                                                            ABM es:
          (x – 2)2+(y – 5)2=10                       (I)
           1
• Dato m =
        L  2
                                         L
                                         A(7; 5)

                        M

Calculamos la ecuación de la recta L .

          y – 5=m (x – 7)
                    L

                1
          y – 5= (x – 7)                             (II)
                2

                                                                                                                  29
Matemática

         ⎛ 3π ⎞                                        En el gráfico se observa que AB= 2 y AM=BM,
A) 2 tan ⎜ ⎟
         ⎝ 8 ⎠                                                                   2
                                                       entonces, AH=HB=            .
   1     ⎛ 3π ⎞                                                                 2
B)   tan ⎜ ⎟
   2     ⎝ 8 ⎠                                         Calculamos la altura MH en el triángulo AHM.
         ⎛ 3π ⎞                                                      2     3π
C) 2 tan ⎜ ⎟
         ⎝ 4 ⎠                                              MH =       tan
                                                                    2      8
     1     ⎛ 3π ⎞                                      Luego
D)     tan ⎜ ⎟
     2     ⎝ 4 ⎠
                                                                 ( AB)(MH )
                                                            S=
         ⎛ 4π ⎞                                                       2
E) 2 tan ⎜    ⎟
         ⎝ 7 ⎠
                                                                       ⎛ 2    3π ⎞
                                                                 ( 2)⎜
                                                                     ⎜   2
                                                                           tan ⎟
                                                                              8 ⎟
                                                            S=         ⎝         ⎠
Solución                                                                 2
Tema
                                                       Por lo tanto,
Circunferencia trigonométrica (C. T.)
                                                                 1    3π
                                                            S=     tan .
Referencias                                                      2    8
• Ubicación de arcos en la C. T.
                                                       Respuesta
• Resolución de triángulos rectángulos.
• Cálculo del área de una región triangular.           El área de la región triangular ABM es igual a
                                                       1     3π
                                                         tan .
Análisis y procedimiento                               2     8

                                               π                                        Alternativa B
Dato: CM = DM → mCM = m DM =
                                               4
                      π π          3π
además, m BM =         +  → m BM =    .
                      2 4          4
                                                       Pregunta N.º 37
                                                       Simplificando la siguiente expresión
                        Y                                 K=sen23Acsc2A+cos23Asec2A+2cos4A,
                 3p          C                         se obtiene
                  4
             M
                                                       A)   6cos22A
                                                       B)   6cos2A
                                               B       C)   8sen2A
         D                                         X   D)   12senA
                                           2
                                                       E)   12cos22A
                         3p               2
                                      H
                          8       2                    Solución
                         A       2
                                                       Tema
                                                       Identidades trigonométricas de arcos múltiples

                                                                                                        30
Matemática

Referencias                                             Solución
• Empleamos las identidades auxiliares del arco         Tema
     triple                                             Funciones trigonométricas
     sen3θ=senθ(2cos2θ+1)
     cos3θ=cosθ(2cos2θ – 1)                             Referencias
• Empleamos la identidad del arco doble relacio-        Para reducir la expresión aplicaremos identidades
     nada con el coseno.                                trigonométricas.
     cos2θ=2cos2θ – 1                                                sen x                    cos x
                                                             tan x =              cot x =
                                                                     cos x                    sen x
Análisis y procedimiento
     K=sen23Acsc2A+cos23Asec2A+2cos4A                   Análisis y procedimiento
                                                                       sen x + tan x             π
entonces                                                    f ( x) =                   x≠K
                                                                       cos x + cot x             2
                    2            2
        ⎛ sen 3 A ⎞  ⎛ cos 3 A ⎞
     K =⎜         ⎟ +⎜         ⎟ + 2 cos 4 A
        ⎝ sen A ⎠    ⎝ cos A ⎠                             cosx+cotx ≠ 0

Ahora aplicamos las identidades del arco triple.           cosx(1+1/senx) ≠ 0
     K=(2cos2A+1)2+(2cos2A – 1)2+2cos4A                    cosx ≠ 0 ∧ senx ≠ – 1
Desarrollando los binomios y aplicando la identi-                                π
                                                           → x ≠ (2n+1)
dad del arco doble, obtenemos                                                    2
     K=2(4cos22A+1)+2(2cos22A – 1)
                                                                             sen x
     → K=12cos22A                                                      sen x +
                                                            f ( x) =         cos x
                                                                             cos x
                                                                     cos x +
Respuesta                                                                    sen x

Entonces, K es igual a 12cos22A.                                           ⎛ 1 + cos x ⎞
                                                                     sen x ⎜           ⎟
                                                                           ⎝ cos x ⎠
                                                            f ( x) =
                                                                           ⎛ 1 + sen x ⎞
                                                                     cos x ⎜           ⎟
                                     Alternativa E                         ⎝ sen x ⎠

                                                                       sen 2 x (1 + cos x )
                                                            f (x) =
                                                                       cos 2 x (1 + sen x )
Pregunta N.º 38                                            senx > – 1
                                                           → 1+senx > 0
              sen x + tan x      π                         cosx > – 1
Sea f ( x ) =               , x≠k .
              cos x + cot x      2                         → 1+cosx > 0
Entonces podemos afirmar que                            Entonces, se deduce que f(x) es positivo.

A)   f(x) toma valores positivos y negativos.
                                                        Respuesta
B)   f(x) toma un número finito de valores negativos.
                                                        f(x) toma solamente valores positivos.
C)   f(x) toma solamente valores negativos.
D)   f(x) toma solamente valores positivos.
                                                                                                      Alternativa D
E)   f(x) es constante.

                                                                                                                      31
Matemática

Pregunta N.º 39                                                                  ⎛ x−y⎞         ⎛ x−y⎞
                                                                       → 4·cos 2 ⎜    ⎟ + 4 cos ⎜    ⎟−3=0
Dado el sistema                                                                  ⎝ 2 ⎠          ⎝ 2 ⎠


     ⎧                4π                                               ⎛       ⎛ x−y⎞     ⎞⎛         ⎛ x−y⎞ ⎞
     ⎪        x+y=                                                     ⎜ 2 cos ⎜    ⎟ + 3 ⎟ ·⎜ 2 cos ⎜    ⎟ − 1⎟ = 0
     ⎨                 3                                               ⎝       ⎝ 2 ⎠      ⎠⎝         ⎝ 2 ⎠ ⎠
     ⎪sec x + sec y = 1
     ⎩
                                                                           ⎛ x−y⎞ 1
                                                                       cos ⎜    ⎟=  o
el valor de cos(x – y) es:                                                 ⎝ 2 ⎠ 2

                                                                              ⎛ x−y⎞   3
         1                     1                      1                   cos ⎜    ⎟=−
A) −                    B) −                 C) −                             ⎝ 2 ⎠    2
         4                     3                      2
                                                                    La ecuación admite para
     1                                            1
D)                                           E)
     4                                            2                        ⎛ x−y⎞ 1
                                                                       cos ⎜    ⎟=
                                                                           ⎝ 2 ⎠ 2
Solución
Tema                                                                Luego, debido a que
Sistemas de ecuaciones trigonométricas
                                                                                               ⎛ x−y⎞
                                                                       cos ( x − y ) = 2 cos 2 ⎜    ⎟ −1
                                                                                               ⎝ 2 ⎠
Referencias
                                                                    Por lo tanto
Transformaciones trigonométricas.
                                                                                           1
                           ⎛ x+y⎞      ⎛ x−y⎞                          cos ( x − y ) = −
     cos x + cos y = 2 cos ⎜    ⎟ ·cos ⎜    ⎟                                              2
                           ⎝ 2 ⎠       ⎝ 2 ⎠
                                                                    Respuesta
Identidad de arco doble.
                                                                                               1
     cos2x=2cos2x – 1                                               El valor de cos(x – y) es − .
                                                                                               2

Análisis y procedimiento
                                                                                                           Alternativa C

De la condición
     secx+secy=1                                                    Pregunta N.º 40
     2 · (cosx+cosy)=2(cosx · cosy)                                 En las circunferencias tangentes de la figura, son
                                                                    datos r0 (radio) y α. Determine el radio R.
             ⎛ x +y⎞      ⎛ x −y⎞
2× 2 ·cos ⎜        ⎟ ·cos ⎜     ⎟ = cos ( x + y ) + cos ( x − y )
             ⎝ 2 ⎠        ⎝ 2 ⎠


Por dato sabemos que

                4π
     x+y=          .
                3
       ⎛ −1 ⎞  ⎛ x−y⎞    1       2⎛ x −y⎞
     4 ⎜ ⎟ cos ⎜    ⎟ = − + 2 cos ⎜     ⎟ −1
       ⎝ 2 ⎠   ⎝ 2 ⎠     2        ⎝ 2 ⎠


                                                                                                                           32
Matemática

   ⎛ 1 − cos α ⎞                             Análisis y procedimiento
A) ⎜           ⎟ r0
   ⎝ cos α ⎠


   ⎛ cos α ⎞                                                                r0
B) ⎜           ⎟ r0
   ⎝ 1 − cos α ⎠

                                                                    R
   ⎛ 1 − cos α ⎞
C) ⎜           ⎟ r0
   ⎝ 1 + cos α ⎠                                                a
                                                                        R

   ⎛ 1 + cos α ⎞                             Por definición tenemos
D) ⎜           ⎟ r0                                        R
   ⎝ cos α ⎠                                     cos α =
                                                         R + r0

                                                Rcosα+r0cosα=R
   ⎛ 1 + cos α ⎞
E) ⎜           ⎟ r0
   ⎝ 1 − cos α ⎠                                r0cosα=R(1 – cosα)
                                                     r cos α
                                                 R= 0
                                                     1 − cos α
Solución
                                                   ⎛ cos α ⎞
Tema                                             R=⎜           ⎟ r0
                                                   ⎝ 1 − cos α ⎠
Razones trigonométricas de un ángulo agudo

                                             Respuesta
Referencias
                                             Entonces, el radio R, en términos de r0 y α, es
Definición del coseno de un ángulo agudo.
                                             ⎛ cos α ⎞
                                             ⎜           ⎟ r0
            cateto adyacente                 ⎝ 1 − cos α ⎠
    cos α =
               hipotenusa                                                        Alternativa B




                                                                                                 33

Contenu connexe

Tendances

SOLUCIONARIO EXAMEN DE ADMISION UNI APTITUD 2009 I
SOLUCIONARIO EXAMEN DE ADMISION UNI APTITUD 2009 ISOLUCIONARIO EXAMEN DE ADMISION UNI APTITUD 2009 I
SOLUCIONARIO EXAMEN DE ADMISION UNI APTITUD 2009 IDANTX
 
Solucionario PRE SAN MARCOS- Semana 16 Ciclo 2016
Solucionario PRE SAN MARCOS- Semana 16 Ciclo 2016 Solucionario PRE SAN MARCOS- Semana 16 Ciclo 2016
Solucionario PRE SAN MARCOS- Semana 16 Ciclo 2016 Mery Lucy Flores M.
 
Solucionario UNI- 2014-2 - Matemática
Solucionario UNI- 2014-2 - MatemáticaSolucionario UNI- 2014-2 - Matemática
Solucionario UNI- 2014-2 - MatemáticaRafael Moreno Yupanqui
 
Problemas resueltos
Problemas resueltosProblemas resueltos
Problemas resueltoselvis CC
 
Solucionario semana nº 5-ordinario 2015-ii
Solucionario semana nº 5-ordinario 2015-iiSolucionario semana nº 5-ordinario 2015-ii
Solucionario semana nº 5-ordinario 2015-iiAldo Martín Livia Reyes
 
Aduni repaso aritmetica 1
Aduni repaso aritmetica 1Aduni repaso aritmetica 1
Aduni repaso aritmetica 1Gerson Quiroz
 
Solucionario del primer examen con ingreso directo de la PRE SAN MARCOS ciclo...
Solucionario del primer examen con ingreso directo de la PRE SAN MARCOS ciclo...Solucionario del primer examen con ingreso directo de la PRE SAN MARCOS ciclo...
Solucionario del primer examen con ingreso directo de la PRE SAN MARCOS ciclo...Mery Lucy Flores M.
 
2011 ii – boletín 19 (nx power-lite) autoevaluandome
2011 ii – boletín 19 (nx power-lite) autoevaluandome2011 ii – boletín 19 (nx power-lite) autoevaluandome
2011 ii – boletín 19 (nx power-lite) autoevaluandometupapitorico
 
Solucionario – cepreunmsm – 2011 ii – boletín 8 – áreas academicas a, d y e
Solucionario – cepreunmsm – 2011 ii – boletín 8 – áreas academicas a, d y eSolucionario – cepreunmsm – 2011 ii – boletín 8 – áreas academicas a, d y e
Solucionario – cepreunmsm – 2011 ii – boletín 8 – áreas academicas a, d y eJazmín Lopez
 
Teoría y problemas de Geometría ADUNI ccesa007
Teoría y problemas de Geometría ADUNI  ccesa007Teoría y problemas de Geometría ADUNI  ccesa007
Teoría y problemas de Geometría ADUNI ccesa007Demetrio Ccesa Rayme
 
Solucionario PRE SAN MARCOS- Semana 7 ciclo 2016 1
Solucionario PRE SAN MARCOS- Semana 7 ciclo 2016 1Solucionario PRE SAN MARCOS- Semana 7 ciclo 2016 1
Solucionario PRE SAN MARCOS- Semana 7 ciclo 2016 1Mery Lucy Flores M.
 
Teoría y problemas de Razonamiento Matemático ADUNI ccesa007
Teoría y problemas de Razonamiento Matemático ADUNI  ccesa007Teoría y problemas de Razonamiento Matemático ADUNI  ccesa007
Teoría y problemas de Razonamiento Matemático ADUNI ccesa007Demetrio Ccesa Rayme
 
Solucionario semana-n-1-ciclo-ordinario-2015-ii
Solucionario semana-n-1-ciclo-ordinario-2015-iiSolucionario semana-n-1-ciclo-ordinario-2015-ii
Solucionario semana-n-1-ciclo-ordinario-2015-iiAldo Martín Livia Reyes
 

Tendances (20)

2010 i semana 4
2010   i semana 42010   i semana 4
2010 i semana 4
 
SOLUCIONARIO EXAMEN DE ADMISION UNI APTITUD 2009 I
SOLUCIONARIO EXAMEN DE ADMISION UNI APTITUD 2009 ISOLUCIONARIO EXAMEN DE ADMISION UNI APTITUD 2009 I
SOLUCIONARIO EXAMEN DE ADMISION UNI APTITUD 2009 I
 
2010 i semana 14
2010   i semana 142010   i semana 14
2010 i semana 14
 
Solucionario PRE SAN MARCOS- Semana 16 Ciclo 2016
Solucionario PRE SAN MARCOS- Semana 16 Ciclo 2016 Solucionario PRE SAN MARCOS- Semana 16 Ciclo 2016
Solucionario PRE SAN MARCOS- Semana 16 Ciclo 2016
 
Solucionario UNI- 2014-2 - Matemática
Solucionario UNI- 2014-2 - MatemáticaSolucionario UNI- 2014-2 - Matemática
Solucionario UNI- 2014-2 - Matemática
 
Semana 07 2016 2
Semana 07 2016 2Semana 07 2016 2
Semana 07 2016 2
 
Problemas resueltos
Problemas resueltosProblemas resueltos
Problemas resueltos
 
Solucionario semana nº 5-ordinario 2015-ii
Solucionario semana nº 5-ordinario 2015-iiSolucionario semana nº 5-ordinario 2015-ii
Solucionario semana nº 5-ordinario 2015-ii
 
Aduni repaso aritmetica 1
Aduni repaso aritmetica 1Aduni repaso aritmetica 1
Aduni repaso aritmetica 1
 
Semana 13 2016 2
Semana 13 2016 2Semana 13 2016 2
Semana 13 2016 2
 
Semana 08 2016 2
Semana 08 2016 2Semana 08 2016 2
Semana 08 2016 2
 
Solucionario del primer examen con ingreso directo de la PRE SAN MARCOS ciclo...
Solucionario del primer examen con ingreso directo de la PRE SAN MARCOS ciclo...Solucionario del primer examen con ingreso directo de la PRE SAN MARCOS ciclo...
Solucionario del primer examen con ingreso directo de la PRE SAN MARCOS ciclo...
 
2010 i semana 9
2010   i semana 92010   i semana 9
2010 i semana 9
 
2011 ii – boletín 19 (nx power-lite) autoevaluandome
2011 ii – boletín 19 (nx power-lite) autoevaluandome2011 ii – boletín 19 (nx power-lite) autoevaluandome
2011 ii – boletín 19 (nx power-lite) autoevaluandome
 
Solucionario – cepreunmsm – 2011 ii – boletín 8 – áreas academicas a, d y e
Solucionario – cepreunmsm – 2011 ii – boletín 8 – áreas academicas a, d y eSolucionario – cepreunmsm – 2011 ii – boletín 8 – áreas academicas a, d y e
Solucionario – cepreunmsm – 2011 ii – boletín 8 – áreas academicas a, d y e
 
Teoría y problemas de Geometría ADUNI ccesa007
Teoría y problemas de Geometría ADUNI  ccesa007Teoría y problemas de Geometría ADUNI  ccesa007
Teoría y problemas de Geometría ADUNI ccesa007
 
Solucionario PRE SAN MARCOS- Semana 7 ciclo 2016 1
Solucionario PRE SAN MARCOS- Semana 7 ciclo 2016 1Solucionario PRE SAN MARCOS- Semana 7 ciclo 2016 1
Solucionario PRE SAN MARCOS- Semana 7 ciclo 2016 1
 
Teoría y problemas de Razonamiento Matemático ADUNI ccesa007
Teoría y problemas de Razonamiento Matemático ADUNI  ccesa007Teoría y problemas de Razonamiento Matemático ADUNI  ccesa007
Teoría y problemas de Razonamiento Matemático ADUNI ccesa007
 
Solucionario semana-n-1-ciclo-ordinario-2015-ii
Solucionario semana-n-1-ciclo-ordinario-2015-iiSolucionario semana-n-1-ciclo-ordinario-2015-ii
Solucionario semana-n-1-ciclo-ordinario-2015-ii
 
Semana 11 2010 ii
Semana 11 2010 iiSemana 11 2010 ii
Semana 11 2010 ii
 

En vedette

Álgebra y Trigonometría - Ejercicios 02
Álgebra y Trigonometría - Ejercicios 02Álgebra y Trigonometría - Ejercicios 02
Álgebra y Trigonometría - Ejercicios 02Giovanni Orozco
 
SOLUCIONARIO EXAMEN DE ADMISION UNI MATEMATICA 2009 I
SOLUCIONARIO EXAMEN DE ADMISION UNI MATEMATICA 2009 ISOLUCIONARIO EXAMEN DE ADMISION UNI MATEMATICA 2009 I
SOLUCIONARIO EXAMEN DE ADMISION UNI MATEMATICA 2009 IDANTX
 
Solucionario examen de la uni 2012 ii matematica-ii
Solucionario examen de la uni 2012 ii matematica-iiSolucionario examen de la uni 2012 ii matematica-ii
Solucionario examen de la uni 2012 ii matematica-iiAmilcar Montalban Sayago
 
Copia de-problemas-de-areas
Copia de-problemas-de-areasCopia de-problemas-de-areas
Copia de-problemas-de-areasJohanna Moscoso
 
Cuaderno matematica sexto_ano (1)
Cuaderno matematica sexto_ano (1)Cuaderno matematica sexto_ano (1)
Cuaderno matematica sexto_ano (1)Bernardita Naranjo
 
Circunferencia proporcionalidad y semejanza
Circunferencia proporcionalidad y semejanzaCircunferencia proporcionalidad y semejanza
Circunferencia proporcionalidad y semejanzaHender Chapoñan
 
Solucionario 3er sumativo cepunt 2010 - i
Solucionario 3er sumativo cepunt   2010 - iSolucionario 3er sumativo cepunt   2010 - i
Solucionario 3er sumativo cepunt 2010 - iErick Vasquez Llanos
 
Guia didáctica el teorema de pitágoras completa
Guia didáctica el teorema de pitágoras completaGuia didáctica el teorema de pitágoras completa
Guia didáctica el teorema de pitágoras completacapachoparra
 
Solucionario del segundo exámen con ingreso directo de la PRE SAN MARCOS cicl...
Solucionario del segundo exámen con ingreso directo de la PRE SAN MARCOS cicl...Solucionario del segundo exámen con ingreso directo de la PRE SAN MARCOS cicl...
Solucionario del segundo exámen con ingreso directo de la PRE SAN MARCOS cicl...Mery Lucy Flores M.
 

En vedette (20)

Matemática UNI 2008-I
Matemática UNI 2008-IMatemática UNI 2008-I
Matemática UNI 2008-I
 
Álgebra y Trigonometría - Ejercicios 02
Álgebra y Trigonometría - Ejercicios 02Álgebra y Trigonometría - Ejercicios 02
Álgebra y Trigonometría - Ejercicios 02
 
SOLUCIONARIO EXAMEN DE ADMISION UNI MATEMATICA 2009 I
SOLUCIONARIO EXAMEN DE ADMISION UNI MATEMATICA 2009 ISOLUCIONARIO EXAMEN DE ADMISION UNI MATEMATICA 2009 I
SOLUCIONARIO EXAMEN DE ADMISION UNI MATEMATICA 2009 I
 
Solucionario examen de la uni 2012 ii matematica-ii
Solucionario examen de la uni 2012 ii matematica-iiSolucionario examen de la uni 2012 ii matematica-ii
Solucionario examen de la uni 2012 ii matematica-ii
 
Plan ADUNI
Plan ADUNIPlan ADUNI
Plan ADUNI
 
Fisica i
Fisica iFisica i
Fisica i
 
Ssm 03 Polinomios
Ssm 03 PolinomiosSsm 03 Polinomios
Ssm 03 Polinomios
 
Areas
AreasAreas
Areas
 
Trigonometría
TrigonometríaTrigonometría
Trigonometría
 
Copia de-problemas-de-areas
Copia de-problemas-de-areasCopia de-problemas-de-areas
Copia de-problemas-de-areas
 
Cuaderno matematica sexto_ano (1)
Cuaderno matematica sexto_ano (1)Cuaderno matematica sexto_ano (1)
Cuaderno matematica sexto_ano (1)
 
Circunferencia proporcionalidad y semejanza
Circunferencia proporcionalidad y semejanzaCircunferencia proporcionalidad y semejanza
Circunferencia proporcionalidad y semejanza
 
Semana 8 alg
Semana 8 algSemana 8 alg
Semana 8 alg
 
3º examen sumativo 2012 iii
3º  examen sumativo 2012 iii3º  examen sumativo 2012 iii
3º examen sumativo 2012 iii
 
Cepunt 2013
Cepunt 2013Cepunt 2013
Cepunt 2013
 
Numeracion 12
Numeracion 12Numeracion 12
Numeracion 12
 
Solucionario 3er sumativo cepunt 2010 - i
Solucionario 3er sumativo cepunt   2010 - iSolucionario 3er sumativo cepunt   2010 - i
Solucionario 3er sumativo cepunt 2010 - i
 
Uni ii fisica 2011
Uni ii fisica 2011Uni ii fisica 2011
Uni ii fisica 2011
 
Guia didáctica el teorema de pitágoras completa
Guia didáctica el teorema de pitágoras completaGuia didáctica el teorema de pitágoras completa
Guia didáctica el teorema de pitágoras completa
 
Solucionario del segundo exámen con ingreso directo de la PRE SAN MARCOS cicl...
Solucionario del segundo exámen con ingreso directo de la PRE SAN MARCOS cicl...Solucionario del segundo exámen con ingreso directo de la PRE SAN MARCOS cicl...
Solucionario del segundo exámen con ingreso directo de la PRE SAN MARCOS cicl...
 

Similaire à SOLUCIONARIO EXAMEN DE ADMISION UNI MATEMATICA II 2009 I

10 EvaluacióN De Control 3 Periodo I
10  EvaluacióN De Control 3   Periodo I10  EvaluacióN De Control 3   Periodo I
10 EvaluacióN De Control 3 Periodo IJuan Galindo
 
SOLUCIONARIO EXAMEN DE ADMISION UNI 2009 - I APTITUD ACADÉMICA - TEMA: S
SOLUCIONARIO EXAMEN DE ADMISION  UNI 2009 - I APTITUD ACADÉMICA - TEMA: SSOLUCIONARIO EXAMEN DE ADMISION  UNI 2009 - I APTITUD ACADÉMICA - TEMA: S
SOLUCIONARIO EXAMEN DE ADMISION UNI 2009 - I APTITUD ACADÉMICA - TEMA: SDANTX
 
Mat ii tema 05 geo puntos rectas y planos
Mat ii tema 05 geo puntos rectas y planosMat ii tema 05 geo puntos rectas y planos
Mat ii tema 05 geo puntos rectas y planosErwin Navarrete
 
Funciones De Dos Variables
Funciones De Dos VariablesFunciones De Dos Variables
Funciones De Dos VariablesJosé Encalada
 
Práctica saint michael matemática de octavo parte 2
Práctica saint michael matemática de octavo parte 2Práctica saint michael matemática de octavo parte 2
Práctica saint michael matemática de octavo parte 2MCMurray
 
Ejercicios Geometría Analítica
Ejercicios Geometría AnalíticaEjercicios Geometría Analítica
Ejercicios Geometría Analítica29121969l
 
Geometria analitica
Geometria analiticaGeometria analitica
Geometria analiticaTzihue Perez
 
funciones cuadraticas y raiz cuadrada.pdf
funciones cuadraticas y raiz cuadrada.pdffunciones cuadraticas y raiz cuadrada.pdf
funciones cuadraticas y raiz cuadrada.pdfmartinmaltez
 
Manual de calculo vectorial 2008
Manual de calculo vectorial 2008Manual de calculo vectorial 2008
Manual de calculo vectorial 2008Frank Mucha
 
Trigonometria
TrigonometriaTrigonometria
Trigonometriagina luz
 

Similaire à SOLUCIONARIO EXAMEN DE ADMISION UNI MATEMATICA II 2009 I (20)

3º examen formativo 2012 iii
3º examen formativo 2012 iii3º examen formativo 2012 iii
3º examen formativo 2012 iii
 
10 EvaluacióN De Control 3 Periodo I
10  EvaluacióN De Control 3   Periodo I10  EvaluacióN De Control 3   Periodo I
10 EvaluacióN De Control 3 Periodo I
 
SOLUCIONARIO EXAMEN DE ADMISION UNI 2009 - I APTITUD ACADÉMICA - TEMA: S
SOLUCIONARIO EXAMEN DE ADMISION  UNI 2009 - I APTITUD ACADÉMICA - TEMA: SSOLUCIONARIO EXAMEN DE ADMISION  UNI 2009 - I APTITUD ACADÉMICA - TEMA: S
SOLUCIONARIO EXAMEN DE ADMISION UNI 2009 - I APTITUD ACADÉMICA - TEMA: S
 
Mat ii tema 05 geo puntos rectas y planos
Mat ii tema 05 geo puntos rectas y planosMat ii tema 05 geo puntos rectas y planos
Mat ii tema 05 geo puntos rectas y planos
 
Funciones De Dos Variables
Funciones De Dos VariablesFunciones De Dos Variables
Funciones De Dos Variables
 
3º examen formativo 2012 iii
3º examen formativo 2012 iii3º examen formativo 2012 iii
3º examen formativo 2012 iii
 
Aplicaciones de la derivada
Aplicaciones de la derivadaAplicaciones de la derivada
Aplicaciones de la derivada
 
Práctica saint michael matemática de octavo parte 2
Práctica saint michael matemática de octavo parte 2Práctica saint michael matemática de octavo parte 2
Práctica saint michael matemática de octavo parte 2
 
Ejercicios Geometría Analítica
Ejercicios Geometría AnalíticaEjercicios Geometría Analítica
Ejercicios Geometría Analítica
 
Derivadas
DerivadasDerivadas
Derivadas
 
Geometria analitica
Geometria analiticaGeometria analitica
Geometria analitica
 
TrigonometríA(I)
TrigonometríA(I)TrigonometríA(I)
TrigonometríA(I)
 
Trigonometría(I)
Trigonometría(I)Trigonometría(I)
Trigonometría(I)
 
Conicas
ConicasConicas
Conicas
 
Funcion cuadratica
Funcion cuadraticaFuncion cuadratica
Funcion cuadratica
 
funciones cuadraticas y raiz cuadrada.pdf
funciones cuadraticas y raiz cuadrada.pdffunciones cuadraticas y raiz cuadrada.pdf
funciones cuadraticas y raiz cuadrada.pdf
 
Examen sumativo
Examen sumativoExamen sumativo
Examen sumativo
 
Manual de calculo vectorial 2008
Manual de calculo vectorial 2008Manual de calculo vectorial 2008
Manual de calculo vectorial 2008
 
Trigonometria
TrigonometriaTrigonometria
Trigonometria
 
U7 t2-ecuaciones-cuadraticas
U7 t2-ecuaciones-cuadraticasU7 t2-ecuaciones-cuadraticas
U7 t2-ecuaciones-cuadraticas
 

Plus de DANTX

DETERMINANTES SOCIALES DE LA DESNUTRICIÓN EN NIÑOS MENORES DE 5 AÑOS ATENDIDO...
DETERMINANTES SOCIALES DE LA DESNUTRICIÓN EN NIÑOS MENORES DE 5 AÑOS ATENDIDO...DETERMINANTES SOCIALES DE LA DESNUTRICIÓN EN NIÑOS MENORES DE 5 AÑOS ATENDIDO...
DETERMINANTES SOCIALES DE LA DESNUTRICIÓN EN NIÑOS MENORES DE 5 AÑOS ATENDIDO...DANTX
 
Formatos de inventario, plantillas, programas y bases de datos
Formatos de inventario, plantillas, programas y bases de datosFormatos de inventario, plantillas, programas y bases de datos
Formatos de inventario, plantillas, programas y bases de datosDANTX
 
DISFUNCIÓN ERÉCTIL
DISFUNCIÓN ERÉCTILDISFUNCIÓN ERÉCTIL
DISFUNCIÓN ERÉCTILDANTX
 
Nutrición y Salud
Nutrición y SaludNutrición y Salud
Nutrición y SaludDANTX
 
Linfomas
Linfomas Linfomas
Linfomas DANTX
 
Ginecomastia Puberal
Ginecomastia  PuberalGinecomastia  Puberal
Ginecomastia PuberalDANTX
 
UN TESORO HISTÓRICO BAJO TIERRA "La vieja santa fe"
UN TESORO HISTÓRICO BAJO TIERRA "La vieja santa fe"UN TESORO HISTÓRICO BAJO TIERRA "La vieja santa fe"
UN TESORO HISTÓRICO BAJO TIERRA "La vieja santa fe"DANTX
 
Canibalismo en china comen bebes y fetos humanos
Canibalismo en china   comen bebes y fetos humanosCanibalismo en china   comen bebes y fetos humanos
Canibalismo en china comen bebes y fetos humanosDANTX
 
Tutorial como ver peliculas online gratis, sin registrarse
Tutorial como ver peliculas online gratis, sin registrarseTutorial como ver peliculas online gratis, sin registrarse
Tutorial como ver peliculas online gratis, sin registrarseDANTX
 
Determinantes sociales de la desnutrición en niños menores de 5 años.
Determinantes sociales de la desnutrición en niños menores de 5 años.Determinantes sociales de la desnutrición en niños menores de 5 años.
Determinantes sociales de la desnutrición en niños menores de 5 años.DANTX
 
ESNI 2009
ESNI 2009ESNI 2009
ESNI 2009DANTX
 
Denotacion & connotacion
Denotacion & connotacionDenotacion & connotacion
Denotacion & connotacionDANTX
 
SOLUCIONARIO EXAMEN DE ADMISION UNI QUIMICA 2009 I
SOLUCIONARIO EXAMEN DE ADMISION UNI QUIMICA 2009 ISOLUCIONARIO EXAMEN DE ADMISION UNI QUIMICA 2009 I
SOLUCIONARIO EXAMEN DE ADMISION UNI QUIMICA 2009 IDANTX
 
SOLUCIONARIO EXAMEN DE ADMISION UNI FISICA 2009 I
SOLUCIONARIO EXAMEN DE ADMISION UNI FISICA 2009 ISOLUCIONARIO EXAMEN DE ADMISION UNI FISICA 2009 I
SOLUCIONARIO EXAMEN DE ADMISION UNI FISICA 2009 IDANTX
 
SOLUCIONARIO EXAMEN DE ADMISION UNI CULTURA 2009 I
SOLUCIONARIO EXAMEN DE ADMISION UNI CULTURA 2009 ISOLUCIONARIO EXAMEN DE ADMISION UNI CULTURA 2009 I
SOLUCIONARIO EXAMEN DE ADMISION UNI CULTURA 2009 IDANTX
 
EXAMEN DE ADMISION UNI QUIMICA 2009 I
EXAMEN DE ADMISION UNI QUIMICA 2009 IEXAMEN DE ADMISION UNI QUIMICA 2009 I
EXAMEN DE ADMISION UNI QUIMICA 2009 IDANTX
 
EXAMEN DE ADMISION UNI MATEMATICA 2009 I
EXAMEN DE ADMISION UNI MATEMATICA 2009 IEXAMEN DE ADMISION UNI MATEMATICA 2009 I
EXAMEN DE ADMISION UNI MATEMATICA 2009 IDANTX
 
EXAMEN DE ADMISION UNI FISICA 2009 I
EXAMEN DE ADMISION UNI FISICA 2009 IEXAMEN DE ADMISION UNI FISICA 2009 I
EXAMEN DE ADMISION UNI FISICA 2009 IDANTX
 
EXAMEN DE ADMISION CULTURA 2009 I
EXAMEN DE ADMISION CULTURA 2009 IEXAMEN DE ADMISION CULTURA 2009 I
EXAMEN DE ADMISION CULTURA 2009 IDANTX
 
EXAMEN DE ADMISION APTITUD ACADEMICA UNI 2009 I
EXAMEN DE ADMISION APTITUD ACADEMICA UNI 2009 IEXAMEN DE ADMISION APTITUD ACADEMICA UNI 2009 I
EXAMEN DE ADMISION APTITUD ACADEMICA UNI 2009 IDANTX
 

Plus de DANTX (20)

DETERMINANTES SOCIALES DE LA DESNUTRICIÓN EN NIÑOS MENORES DE 5 AÑOS ATENDIDO...
DETERMINANTES SOCIALES DE LA DESNUTRICIÓN EN NIÑOS MENORES DE 5 AÑOS ATENDIDO...DETERMINANTES SOCIALES DE LA DESNUTRICIÓN EN NIÑOS MENORES DE 5 AÑOS ATENDIDO...
DETERMINANTES SOCIALES DE LA DESNUTRICIÓN EN NIÑOS MENORES DE 5 AÑOS ATENDIDO...
 
Formatos de inventario, plantillas, programas y bases de datos
Formatos de inventario, plantillas, programas y bases de datosFormatos de inventario, plantillas, programas y bases de datos
Formatos de inventario, plantillas, programas y bases de datos
 
DISFUNCIÓN ERÉCTIL
DISFUNCIÓN ERÉCTILDISFUNCIÓN ERÉCTIL
DISFUNCIÓN ERÉCTIL
 
Nutrición y Salud
Nutrición y SaludNutrición y Salud
Nutrición y Salud
 
Linfomas
Linfomas Linfomas
Linfomas
 
Ginecomastia Puberal
Ginecomastia  PuberalGinecomastia  Puberal
Ginecomastia Puberal
 
UN TESORO HISTÓRICO BAJO TIERRA "La vieja santa fe"
UN TESORO HISTÓRICO BAJO TIERRA "La vieja santa fe"UN TESORO HISTÓRICO BAJO TIERRA "La vieja santa fe"
UN TESORO HISTÓRICO BAJO TIERRA "La vieja santa fe"
 
Canibalismo en china comen bebes y fetos humanos
Canibalismo en china   comen bebes y fetos humanosCanibalismo en china   comen bebes y fetos humanos
Canibalismo en china comen bebes y fetos humanos
 
Tutorial como ver peliculas online gratis, sin registrarse
Tutorial como ver peliculas online gratis, sin registrarseTutorial como ver peliculas online gratis, sin registrarse
Tutorial como ver peliculas online gratis, sin registrarse
 
Determinantes sociales de la desnutrición en niños menores de 5 años.
Determinantes sociales de la desnutrición en niños menores de 5 años.Determinantes sociales de la desnutrición en niños menores de 5 años.
Determinantes sociales de la desnutrición en niños menores de 5 años.
 
ESNI 2009
ESNI 2009ESNI 2009
ESNI 2009
 
Denotacion & connotacion
Denotacion & connotacionDenotacion & connotacion
Denotacion & connotacion
 
SOLUCIONARIO EXAMEN DE ADMISION UNI QUIMICA 2009 I
SOLUCIONARIO EXAMEN DE ADMISION UNI QUIMICA 2009 ISOLUCIONARIO EXAMEN DE ADMISION UNI QUIMICA 2009 I
SOLUCIONARIO EXAMEN DE ADMISION UNI QUIMICA 2009 I
 
SOLUCIONARIO EXAMEN DE ADMISION UNI FISICA 2009 I
SOLUCIONARIO EXAMEN DE ADMISION UNI FISICA 2009 ISOLUCIONARIO EXAMEN DE ADMISION UNI FISICA 2009 I
SOLUCIONARIO EXAMEN DE ADMISION UNI FISICA 2009 I
 
SOLUCIONARIO EXAMEN DE ADMISION UNI CULTURA 2009 I
SOLUCIONARIO EXAMEN DE ADMISION UNI CULTURA 2009 ISOLUCIONARIO EXAMEN DE ADMISION UNI CULTURA 2009 I
SOLUCIONARIO EXAMEN DE ADMISION UNI CULTURA 2009 I
 
EXAMEN DE ADMISION UNI QUIMICA 2009 I
EXAMEN DE ADMISION UNI QUIMICA 2009 IEXAMEN DE ADMISION UNI QUIMICA 2009 I
EXAMEN DE ADMISION UNI QUIMICA 2009 I
 
EXAMEN DE ADMISION UNI MATEMATICA 2009 I
EXAMEN DE ADMISION UNI MATEMATICA 2009 IEXAMEN DE ADMISION UNI MATEMATICA 2009 I
EXAMEN DE ADMISION UNI MATEMATICA 2009 I
 
EXAMEN DE ADMISION UNI FISICA 2009 I
EXAMEN DE ADMISION UNI FISICA 2009 IEXAMEN DE ADMISION UNI FISICA 2009 I
EXAMEN DE ADMISION UNI FISICA 2009 I
 
EXAMEN DE ADMISION CULTURA 2009 I
EXAMEN DE ADMISION CULTURA 2009 IEXAMEN DE ADMISION CULTURA 2009 I
EXAMEN DE ADMISION CULTURA 2009 I
 
EXAMEN DE ADMISION APTITUD ACADEMICA UNI 2009 I
EXAMEN DE ADMISION APTITUD ACADEMICA UNI 2009 IEXAMEN DE ADMISION APTITUD ACADEMICA UNI 2009 I
EXAMEN DE ADMISION APTITUD ACADEMICA UNI 2009 I
 

Dernier

MAYO 1 PROYECTO día de la madre el amor más grande
MAYO 1 PROYECTO día de la madre el amor más grandeMAYO 1 PROYECTO día de la madre el amor más grande
MAYO 1 PROYECTO día de la madre el amor más grandeMarjorie Burga
 
Planificacion Anual 4to Grado Educacion Primaria 2024 Ccesa007.pdf
Planificacion Anual 4to Grado Educacion Primaria   2024   Ccesa007.pdfPlanificacion Anual 4to Grado Educacion Primaria   2024   Ccesa007.pdf
Planificacion Anual 4to Grado Educacion Primaria 2024 Ccesa007.pdfDemetrio Ccesa Rayme
 
Registro Auxiliar - Primaria 2024 (1).pptx
Registro Auxiliar - Primaria  2024 (1).pptxRegistro Auxiliar - Primaria  2024 (1).pptx
Registro Auxiliar - Primaria 2024 (1).pptxFelicitasAsuncionDia
 
FORTI-MAYO 2024.pdf.CIENCIA,EDUCACION,CULTURA
FORTI-MAYO 2024.pdf.CIENCIA,EDUCACION,CULTURAFORTI-MAYO 2024.pdf.CIENCIA,EDUCACION,CULTURA
FORTI-MAYO 2024.pdf.CIENCIA,EDUCACION,CULTURAEl Fortí
 
EXPANSIÓN ECONÓMICA DE OCCIDENTE LEÓN.pptx
EXPANSIÓN ECONÓMICA DE OCCIDENTE LEÓN.pptxEXPANSIÓN ECONÓMICA DE OCCIDENTE LEÓN.pptx
EXPANSIÓN ECONÓMICA DE OCCIDENTE LEÓN.pptxPryhaSalam
 
ACUERDO MINISTERIAL 078-ORGANISMOS ESCOLARES..pptx
ACUERDO MINISTERIAL 078-ORGANISMOS ESCOLARES..pptxACUERDO MINISTERIAL 078-ORGANISMOS ESCOLARES..pptx
ACUERDO MINISTERIAL 078-ORGANISMOS ESCOLARES..pptxzulyvero07
 
cortes de luz abril 2024 en la provincia de tungurahua
cortes de luz abril 2024 en la provincia de tungurahuacortes de luz abril 2024 en la provincia de tungurahua
cortes de luz abril 2024 en la provincia de tungurahuaDANNYISAACCARVAJALGA
 
SEXTO SEGUNDO PERIODO EMPRENDIMIENTO.pptx
SEXTO SEGUNDO PERIODO EMPRENDIMIENTO.pptxSEXTO SEGUNDO PERIODO EMPRENDIMIENTO.pptx
SEXTO SEGUNDO PERIODO EMPRENDIMIENTO.pptxYadi Campos
 
Curso = Metodos Tecnicas y Modelos de Enseñanza.pdf
Curso = Metodos Tecnicas y Modelos de Enseñanza.pdfCurso = Metodos Tecnicas y Modelos de Enseñanza.pdf
Curso = Metodos Tecnicas y Modelos de Enseñanza.pdfFrancisco158360
 
TEMA 13 ESPAÑA EN DEMOCRACIA:DISTINTOS GOBIERNOS
TEMA 13 ESPAÑA EN DEMOCRACIA:DISTINTOS GOBIERNOSTEMA 13 ESPAÑA EN DEMOCRACIA:DISTINTOS GOBIERNOS
TEMA 13 ESPAÑA EN DEMOCRACIA:DISTINTOS GOBIERNOSjlorentemartos
 
30-de-abril-plebiscito-1902_240420_104511.pdf
30-de-abril-plebiscito-1902_240420_104511.pdf30-de-abril-plebiscito-1902_240420_104511.pdf
30-de-abril-plebiscito-1902_240420_104511.pdfgimenanahuel
 
Plan Refuerzo Escolar 2024 para estudiantes con necesidades de Aprendizaje en...
Plan Refuerzo Escolar 2024 para estudiantes con necesidades de Aprendizaje en...Plan Refuerzo Escolar 2024 para estudiantes con necesidades de Aprendizaje en...
Plan Refuerzo Escolar 2024 para estudiantes con necesidades de Aprendizaje en...Carlos Muñoz
 
ACERTIJO DE LA BANDERA OLÍMPICA CON ECUACIONES DE LA CIRCUNFERENCIA. Por JAVI...
ACERTIJO DE LA BANDERA OLÍMPICA CON ECUACIONES DE LA CIRCUNFERENCIA. Por JAVI...ACERTIJO DE LA BANDERA OLÍMPICA CON ECUACIONES DE LA CIRCUNFERENCIA. Por JAVI...
ACERTIJO DE LA BANDERA OLÍMPICA CON ECUACIONES DE LA CIRCUNFERENCIA. Por JAVI...JAVIER SOLIS NOYOLA
 
Caja de herramientas de inteligencia artificial para la academia y la investi...
Caja de herramientas de inteligencia artificial para la academia y la investi...Caja de herramientas de inteligencia artificial para la academia y la investi...
Caja de herramientas de inteligencia artificial para la academia y la investi...Lourdes Feria
 
Historia y técnica del collage en el arte
Historia y técnica del collage en el arteHistoria y técnica del collage en el arte
Historia y técnica del collage en el arteRaquel Martín Contreras
 
Heinsohn Privacidad y Ciberseguridad para el sector educativo
Heinsohn Privacidad y Ciberseguridad para el sector educativoHeinsohn Privacidad y Ciberseguridad para el sector educativo
Heinsohn Privacidad y Ciberseguridad para el sector educativoFundación YOD YOD
 

Dernier (20)

MAYO 1 PROYECTO día de la madre el amor más grande
MAYO 1 PROYECTO día de la madre el amor más grandeMAYO 1 PROYECTO día de la madre el amor más grande
MAYO 1 PROYECTO día de la madre el amor más grande
 
Planificacion Anual 4to Grado Educacion Primaria 2024 Ccesa007.pdf
Planificacion Anual 4to Grado Educacion Primaria   2024   Ccesa007.pdfPlanificacion Anual 4to Grado Educacion Primaria   2024   Ccesa007.pdf
Planificacion Anual 4to Grado Educacion Primaria 2024 Ccesa007.pdf
 
Registro Auxiliar - Primaria 2024 (1).pptx
Registro Auxiliar - Primaria  2024 (1).pptxRegistro Auxiliar - Primaria  2024 (1).pptx
Registro Auxiliar - Primaria 2024 (1).pptx
 
FORTI-MAYO 2024.pdf.CIENCIA,EDUCACION,CULTURA
FORTI-MAYO 2024.pdf.CIENCIA,EDUCACION,CULTURAFORTI-MAYO 2024.pdf.CIENCIA,EDUCACION,CULTURA
FORTI-MAYO 2024.pdf.CIENCIA,EDUCACION,CULTURA
 
EXPANSIÓN ECONÓMICA DE OCCIDENTE LEÓN.pptx
EXPANSIÓN ECONÓMICA DE OCCIDENTE LEÓN.pptxEXPANSIÓN ECONÓMICA DE OCCIDENTE LEÓN.pptx
EXPANSIÓN ECONÓMICA DE OCCIDENTE LEÓN.pptx
 
ACUERDO MINISTERIAL 078-ORGANISMOS ESCOLARES..pptx
ACUERDO MINISTERIAL 078-ORGANISMOS ESCOLARES..pptxACUERDO MINISTERIAL 078-ORGANISMOS ESCOLARES..pptx
ACUERDO MINISTERIAL 078-ORGANISMOS ESCOLARES..pptx
 
cortes de luz abril 2024 en la provincia de tungurahua
cortes de luz abril 2024 en la provincia de tungurahuacortes de luz abril 2024 en la provincia de tungurahua
cortes de luz abril 2024 en la provincia de tungurahua
 
Power Point: Fe contra todo pronóstico.pptx
Power Point: Fe contra todo pronóstico.pptxPower Point: Fe contra todo pronóstico.pptx
Power Point: Fe contra todo pronóstico.pptx
 
SEXTO SEGUNDO PERIODO EMPRENDIMIENTO.pptx
SEXTO SEGUNDO PERIODO EMPRENDIMIENTO.pptxSEXTO SEGUNDO PERIODO EMPRENDIMIENTO.pptx
SEXTO SEGUNDO PERIODO EMPRENDIMIENTO.pptx
 
Curso = Metodos Tecnicas y Modelos de Enseñanza.pdf
Curso = Metodos Tecnicas y Modelos de Enseñanza.pdfCurso = Metodos Tecnicas y Modelos de Enseñanza.pdf
Curso = Metodos Tecnicas y Modelos de Enseñanza.pdf
 
TEMA 13 ESPAÑA EN DEMOCRACIA:DISTINTOS GOBIERNOS
TEMA 13 ESPAÑA EN DEMOCRACIA:DISTINTOS GOBIERNOSTEMA 13 ESPAÑA EN DEMOCRACIA:DISTINTOS GOBIERNOS
TEMA 13 ESPAÑA EN DEMOCRACIA:DISTINTOS GOBIERNOS
 
30-de-abril-plebiscito-1902_240420_104511.pdf
30-de-abril-plebiscito-1902_240420_104511.pdf30-de-abril-plebiscito-1902_240420_104511.pdf
30-de-abril-plebiscito-1902_240420_104511.pdf
 
Plan Refuerzo Escolar 2024 para estudiantes con necesidades de Aprendizaje en...
Plan Refuerzo Escolar 2024 para estudiantes con necesidades de Aprendizaje en...Plan Refuerzo Escolar 2024 para estudiantes con necesidades de Aprendizaje en...
Plan Refuerzo Escolar 2024 para estudiantes con necesidades de Aprendizaje en...
 
ACERTIJO DE LA BANDERA OLÍMPICA CON ECUACIONES DE LA CIRCUNFERENCIA. Por JAVI...
ACERTIJO DE LA BANDERA OLÍMPICA CON ECUACIONES DE LA CIRCUNFERENCIA. Por JAVI...ACERTIJO DE LA BANDERA OLÍMPICA CON ECUACIONES DE LA CIRCUNFERENCIA. Por JAVI...
ACERTIJO DE LA BANDERA OLÍMPICA CON ECUACIONES DE LA CIRCUNFERENCIA. Por JAVI...
 
Caja de herramientas de inteligencia artificial para la academia y la investi...
Caja de herramientas de inteligencia artificial para la academia y la investi...Caja de herramientas de inteligencia artificial para la academia y la investi...
Caja de herramientas de inteligencia artificial para la academia y la investi...
 
Historia y técnica del collage en el arte
Historia y técnica del collage en el arteHistoria y técnica del collage en el arte
Historia y técnica del collage en el arte
 
Heinsohn Privacidad y Ciberseguridad para el sector educativo
Heinsohn Privacidad y Ciberseguridad para el sector educativoHeinsohn Privacidad y Ciberseguridad para el sector educativo
Heinsohn Privacidad y Ciberseguridad para el sector educativo
 
Fe contra todo pronóstico. La fe es confianza.
Fe contra todo pronóstico. La fe es confianza.Fe contra todo pronóstico. La fe es confianza.
Fe contra todo pronóstico. La fe es confianza.
 
Tema 8.- PROTECCION DE LOS SISTEMAS DE INFORMACIÓN.pdf
Tema 8.- PROTECCION DE LOS SISTEMAS DE INFORMACIÓN.pdfTema 8.- PROTECCION DE LOS SISTEMAS DE INFORMACIÓN.pdf
Tema 8.- PROTECCION DE LOS SISTEMAS DE INFORMACIÓN.pdf
 
Sesión de clase: Fe contra todo pronóstico
Sesión de clase: Fe contra todo pronósticoSesión de clase: Fe contra todo pronóstico
Sesión de clase: Fe contra todo pronóstico
 

SOLUCIONARIO EXAMEN DE ADMISION UNI MATEMATICA II 2009 I

  • 1. Matemática Pregunta N.º 20 Ejecución del plan I. Sea p(x) el polinomio de grado n, donde n es el menor posible y cuya gráfica se representa a continuación. p(x)=k(x – 1)2a(x – 2)2b – 1; a, b ∈ Z+ Encuentre el residuo al efectuar la división de Como el grado de p(x) es el menor posible, p(x) con q(x)=x – 3. entonces a=1 y A) – 6 b=1 B) – 4 Luego, tenemos C) – 1 p(x)=k(x – 1)2(x – 2) D) 1 De la gráfica E) 4 p(0)=2 Solución p(0)=k(–1)2(–2) Tema p(0)=2 → k=–1 Gráfica de funciones polinomiales Luego p(x)=–(x – 1)2(x – 2) Referencias II. Aplicando el teorema del resto tenemos Para la solución del problema se necesita p( x) conocer: x−3 • Gráfica de una función polinomial. → R(x)=p(3) • Teorema del resto. p(3)=–(2)2(1) Análisis y procedimiento ∴ p(3)=– 4 Plan de resolución Respuesta I. A partir de la gráfica, hallar la regla de correspondencia de p(x). El residuo de dividir p(x) entre x – 3 es – 4. II. Aplicar el teorema del resto. Alternativa B 15
  • 2. Matemática Pregunta N.º 21 En la figura mostrada ABCD es un cuadrado de lado 2R, además BC es diámetro de la semicircun- ferencia de centro O y radio de longitud R. Si T es un punto de tangencia entonces m TOA es Como ABCD es un cuadrado → BC=CD=2(BO)=2(OC)=2R Trazamos OD → OD: Bisectriz del CDT 53º Luego, OCD (not. ): 2 53º m CDO= y A) 7,5 2 B) 8 53º m ODT= C) 10 2 D) 10,5 En TOCD: inscriptible E) 12,5 → m BOT=m CDT m BOT=53º Solución 53º Tema OBA (not ) 2 Circunferencia 53º → m BAO= 2 Referencias En OBA En la pregunta nos piden la medida de un ángulo; 53º entonces, debemos ubicarlo en una figura donde 53º+x+ =90º 2 se puede obtener dicha medida; por ejemplo, 21º un triángulo; además, como se observa una x= 2 semicircunferencia debemos aplicar los teoremas → x=10,5º que se cumplen en la circunferencia. Respuesta Análisis y procedimiento La medida del ángulo TOA es 10,5º. En el gráfico, nos piden x. Alternativa D 16
  • 3. Matemática Pregunta N.º 22 QR // DB ABC es un triángulo rectángulo. Exteriormente a BD los catetos se construyen los triángulos equiláteros → m RQC=150º y RQ= 2 ABD y BEC. P, Q y R son puntos medios de BE, BC y DC respectivamente. Si el área de la región PQ // EC triangular ABC es 32 cm2, entonces el área de la → m PQC=120º y región triangular PQR (en cm2) es EC PQ= 2 A) 4 B) 6 C) 8 D) 12 E) 16 Luego m PQR=90º Solución En el gráfico, Tema PQR ~ ABC (caso LAL de razón 1/2) Área de regiones triangulares Por áreas de regiones semejantes 2 Referencias A PQR ⎛ razón de ⎞ =⎜ ⎟ Para relacionar las áreas de dos regiones trian- A ABC ⎝ semejanza ⎠ gulares, se busca la relación entre los elementos Reemplazamos de ambos triángulos (lados, alturas, medida de 2 ángulos, etc.). A PQR ⎛1⎞ =⎜ ⎟ 32 ⎝ 2⎠ Análisis y procedimiento → APQR=8 Respuesta El área de la región triangular PQR (en cm2) es 8. Alternativa C Pregunta N.º 23 Indique la secuencia correcta después de determi- nar si la proposición es verdadera (V) o falsa (F). I. Si dos planos son perpendiculares a dos rectas Piden APQR: área de la región triangular PQR. diferentes que se intersectan, entonces dichos Dato A ABC: área de la región triangular ABC. planos también se intersectan. (A ABC=32) II. El lugar geométrico que determinan los pies de los segmentos oblicuos de longitudes iguales Por ser P, Q y R puntos medios, se determinan trazadas desde un punto exterior a un plano bases medias en los triángulos BEC y DBC. es una circunferencia. 17
  • 4. Matemática III. Toda recta es perpendicular a un plano, si es II. ortogonal a dos rectas diferentes no paralelas contenidas en dicho plano. A) VVF B) VFV C) FFV D) VVV E) FFF Solución • Como el punto Q es exterior al plano, traza- Tema mos QQ' de modo que Q' sea la proyección Geometría del espacio. Rectas y planos ortogonal de Q sobre el plano W. • En el gráfico, los triángulos rectángulos Referencias AQ'Q; BQ'Q y DQ'Q son congruentes En este tipo de preguntas debemos hacer una entre sí. comparación entre los conceptos teóricos y los • Luego, m=n=p=… casos posibles que plantean las proposiciones. De • Además, el punto Q' equidista de A, B, esta manera, determinamos la veracidad o falsedad C, D, … de la proposición dada. Por lo tanto, el lugar geométrico que deter- minan A, B, C y D es una circunferencia de Análisis y procedimiento centro Q'. Esta pregunta consta de tres proposiciones. Entonces, la proposición es verdadera. I. En el espacio, solo se admiten dos posiciones III. En el gráfico, para que una recta sea perpendicular relativas entre dos planos: son paralelos o son a un plano, debe ser perpendicular a dos rectas secantes. no paralelas contenidas en dicho plano. Entonces, la proposición es verdadera. • En la fig.1, los planos son paralelos si son perpendiculares a una misma recta. Respuesta • En la fig. 2, los planos son secantes si son La secuencia correcta después de analizar las perpendiculares a dos rectas que se interse- proposiciones es VVV. can (proposición de la pregunta). Entonces, la proposición es verdadera. Alternativa D 18
  • 5. Matemática Pregunta N.º 24 Análisis y procedimiento En la figura mostrada, ABCD es un trapecio Piden rectángulo tal que CD=BC=2AB=2a. Si PQ es Volumen de la pirámide Q-BCP: perpendicular al plano del trapecio tal que PQ=a 1 y los volúmenes de las pirámides Q-ABP y Q-CDP Vx = [ A BCP ][PQ] (I) 3 son iguales, calcule el volumen de la pirámide Q-BCP. Del gráfico tenemos PQ=a (II) Como los volúmenes de las pirámides Q-ABP y Q-PCD son iguales, al tener la misma altura, las A) 1 a 3 B) 3 a 3 áreas de sus bases son también iguales. 2 8 Entonces, AABP=ACPD=4A. 4 3 C) a 5 En el plano de la base 7 3 5 3 D) a E) a 8 9 Solución Tema Geometría del espacio. Pirámide Referencias En preguntas donde piden el cálculo o la relación de volúmenes, conviene hacer un análisis de las Del dato de áreas iguales → AP=2(PD) longitudes de las alturas o de las relaciones de Por relación de áreas, el área de la región trapecial: las bases. Generalmente, para el cálculo del área de la base se emplean capítulos anteriores de ⎛ a + 2a ⎞ a2 18 A = ⎜ ⎟ (2a) → A = geometría plana. ⎝ 2 ⎠ 6 19
  • 6. Matemática Luego, 5a 2 ABCP=10A= (III) 3 Reemplazamos (II) y (III) en (I) 1 ⎛ 5a 3 ⎞ 5a 3 → Vx= ⎜ ⎟ (a) = 3⎝ 3 ⎠ 9 y para poder aprovechar el ángulo de inclinación Respuesta es preciso asociarlo con el teorema de las tres 5a 3 perpendiculares. El volumen de la pirámide Q-BCP es 9 Análisis y procedimiento Alternativa E Graficamos el prisma según las condiciones planteadas. Pregunta N.º 25 D' C' M La altura de un prisma recto mide 1 u, su base es S' B' una región limitada por un rombo cuyo lado mide 1u N A' 2 u y su ángulo agudo mide 30º. Por un lado de 2u C h 1u la base se traza un plano que interseca al prisma 30º 2u D y está inclinado un ángulo de 60º con respecto 2u S 60º 30º B de la base, luego el área de la sección (en u2) que H A 3u resulta en el prisma es: 2u 5 4 A) 2 3 B) C) donde ABCD es un rombo de lado 2 u y la 3 3 m ABC=30º. 3 2 D) E) 3 3 Si trazamos CH ⊥ AB ... 1.a ⊥ Solución SS' ⊥ CH ... 2.a ⊥ Tema → S'H ⊥ AB ... 3.a ⊥ Prisma Sea S'H=h. Referencias Como la altura del prisma es 1 u → S'S=1 u Al trazar planos secantes a un sólido, este determina Luego, en el S'SH: secciones planas, que varían de acuerdo al ángulo de inclinación y el lugar por donde interseca. Así, hsen60º=1 u un plano secante en un prisma puede determinar 2 → h= u una sección triangular, cuadrangular, ... 3 20
  • 7. Matemática Luego, el área de la sección ABMN, que es una Análisis y procedimiento región paralelográmica, se calcula multiplicando Piden r2+3r. AB y h. Las longitudes de los lados del polígono convexo de 8 lados están en progresión geométrica de ⎛ 2 ⎞ ABMN= AB h = ( 2 u ) ⎜ A ( ) u⎟ razón r. ⎝ 3 ⎠ B a ar 4 2 A C A ABMN = u 3 ar7 ar2 Respuesta H D 4 El área de la sección en u2 es . 3 ar3 ar6 Alternativa C G E ar5 ar4 F Pregunta N.º 26 además Se tiene un polígono convexo de 8 lados circuns- AB= ´ 1 , BC= ´ 2 , CD= ´ 3 , DE= ´ 4 , EF= ´ 5 , crita a una circunferencia, si las longitudes de sus FG=´6, GH=´7 y HA=´8, lados están en progresión geométrica de razón r. En el octógono circunscrito por el teorema de Pithot Determine r2+3r. general, tenemos: A) 1 B) 4 C) 10 ´1+´3+´5+´7=´2+´4+´6+´8 D) 18 E) 28 → a+ar2+ar4+ar6=ar+ar3+ar5+ar7 Factorizamos Solución a(1+r2+r4+r6)=ar(1+r2+r4+r6) Tema → r=1 Polígonos circunscritos a una circunferencia: Teorema de Pithot generalizado Respuesta El valor de r2+3r es 4. Referencias En un cuadrilátero circunscrito o circunscriptible, Alternativa B se cumple el teorema de Pithot, es decir, la suma de longitudes de lados opuestos son iguales. En un polígono circunscrito o circunscriptible se Pregunta N.º 27 cumple que la suma de longitudes de lugar par es igual a la suma de longitudes de lugar impar, Se da un triángulo ABC cuyos lados AB y BC es considerado para un cuadrilátero, hexágono, miden 8 m y 6 m respectivamente. Sobre AB octógono, ..., en polígonos cuyo número de lados se toma el punto D. Si m BAC=m BCD. es par. Entonces AD es: 21
  • 8. Matemática A) 3,5 B) 4 C) 4,5 Piden AD D) 5 E) 5,5 Datos: AB=8, BC=6 Solución m BAC=m BCD Tema ABC: Por teorema de semejanza Semejanza de triángulos tenemos: (BC)2=(AB)(BD) (I ) también Referencias BD=8 – AD Cuando en un triángulo se desea relacionar las Reemplazamos: longitudes de lados y segmentos determinados 62=8(8 – AD) por una ceviana, se puede recurrir a la teoría de → AD=3,5 semejanza, y más aún si la medida de un ángulo Respuesta es igual al ángulo determinado por dicha ceviana Entonces, AD es 3,5. y un lado; por ejemplo: Alternativa A B q x Pregunta N.º 28 En figura, AB y AC con diámetros, CT es tan- gente al arco AB, AB=BC=2r y ET=4. Calcule r. C q M m A b Teorema: En el ABC m BAC=m MBC=θ A) 2 3 B) 2 2 C) 3 → x2=bm D) 6 E) 3 3 Análisis y procedimiento B Solución Tema Semejanza de triángulos 8D 6 Referencias En el problema nos piden calcular el radio de la q semicircunferencia menor, para ello debemos rela- q A C cionar el dato numérico con la variable, utilizando 22
  • 9. Matemática los teoremas que se cumplen en circunferencias Pregunta N.º 29 tangentes interiores. Luego, para obtener el valor En un triángulo ABC se cumple AB=2 m y del radio debemos establecer una operación que AC=32 m. Halle el perímetro del triángulo en relacione la incógnita con los datos. metros, sabiendo que es un número entero y el ángulo en A es obtuso. Análisis y procedimiento E A) 65 B) 66 C) 67 D) 68 E) 69 D 4 4 2 T 4 a Solución 2 2 a a Tema a A r r B 2r C Clasificación de triángulos: Triángulo obstusángulo. Trazamos BT → m BTA=90º Referencias Por teorema Para realizar el cálculo del perímetro, es necesario ET=TA=4 conocer BC, el cual, por dato, debe ser entero. Como las longitudes de los otros dos lados son Trazamos AD conocidas, podemos restringir a BC mediante el → AT es bisectriz del DAC teorema de existencia; pero como la medida de m DAT=m TAC=α un ángulo interior es mayor de 90º (obtuso), se Luego puede realizar la restricción de BC por la naturaleza del triángulo. m ECD=m DAE=α En AEC: Teorema de semejanza Análisis y procedimiento (EC)2=(8)(4) Por dato del problema tenemos → EC = 4 2 AB=2, AEC: Teorema base media AC=32 y → TB = 2 2 m BAC>90º 2 ATB: (2r)2=42+( 2 2 ) Piden r= 6 2P ABC=2+32+BC=34+BC. Respuesta B El valor de r es 6. 2 C Alternativa D 32 A 23
  • 10. Matemática En el ABC: Existencia de triángulos 27 54 108 A) B) C) π π π 32 – 2 < BC < 32+2 (I) • Como m BAC>90º D) 54 E) 108 322+22 < BC2 32,06 < BC (II) Solución • Luego, relacionamos las restricciones (I) y (II). Tema 32,06 < BC < 34 (III) Sólidos geométricos • 2P ABC=34+BC Como el perímetro es entero, entonces, BC es Referencias entero. Para calcular el volumen de una pirámide se ne- • Luego, de la expresión (III) obtenemos cesita conocer el área de su base y la altura de la BC=33 pirámide, mientras que para calcular el volumen del cilindro se requiere conocer el área de su base ∴ 2P ABC=67 y su altura. Como el cilindro es circular oblicuo, su base es un círculo, mientras que la base de la Respuesta pirámide es un triángulo equilátero. El perímetro de la región triangular ABC en metros es 67. Análisis y procedimiento Del gráfico que nos dan como dato podemos no- Alternativa C tar que ambos sólidos tienen la misma altura y el triángulo de la base de la pirámide está inscrita en la circunferencia que limita la base del cilindro. Pregunta N.º 30 Denotemos los vértices de la base de la pirámide En la figura se tiene una pirámide inscrita en un como A, B y C, y r el radio del círculo de la base del cilindro. cilindro circular oblicuo. La base de la pirámide es un triángulo equilátero. El volumen de la O 27 3 r pirámide es cm3. Calcule el volumen del π cilindro (en cm3). B A r C Graficando el triángulo equilátero inscrito en la circunferencia tenemos: 24
  • 11. Matemática B Pregunta N.º 31 30º 30º En un polígono convexo equiángulo ABCDEF se r r tiene AB=7, CD=6 y DE=8. Calcule BF. O' 7 r r A) 3 B) 7 C) 5 3 120º 2 A C r 2 D) 7 2 E) 7 3 En el AO'C: Solución Tema AO=r=OC Polígonos m AOC=120º → AC=r 3=AB=BC Referencias Ahora podemos calcular el volumen de la pirá- Dentro del grupo de los polígonos tenemos al mide. polígono equiángulo, que se caracteriza por que sus medidas angulares internas y externas son, 2 1 1 ⎛ (r 3 ) 3 ⎞ respectivamente, iguales. VO-ABC= (Abase)×h= ⎜⎜ ⎟×h ⎟ Como se conoce que la suma de las medidas 3 3⎝ 4 ⎠ angulares de un polígono convexo es 180º(n – 2) 2 y n es el número de lados, entonces, la medida de VO-ABC= r 3⋅ h= 27 3 cm3 un ángulo interior será: 4 π De aquí podemos despejar las variables y obte- 180º ( n − 2 ) i= nemos: n πr 2 · h=108 cm3 (I) Análisis y procedimiento Ahora calculamos el volumen del cilindro Según el dato del problema, el polígono equián- Vcilindro=A base×h gulo es ABCDEF, es decir, tiene seis lados (n=6); Vcilindro=πr 2×h entonces, De (I): Vcilindro=108 cm3 180º ( 6 − 2 ) i( 6 ) = = 120º. 6 Respuesta Grafiquemos el hexágono con las condiciones del El volumen del cilindro en cm3 es 108. problema: AB=7, CD=6 y Alternativa E DE=8. 25
  • 12. Matemática B C Pregunta N.º 32 120º El ángulo de desarrollo de un cono circular recto 7 6 mide 120º. Si la altura del cono mide 4 cm, A 120º 120º D entonces el radio (en cm) del cono es: x 60º 60º a a 8 8 2 A) B) 2 C) 3 60º 60º 120º 60º 60º 2 M a F E 8 N D) 2 2 E) 2 3 Al prolongar los lados BA, EF y CD, las medidas de los ángulos externos en A, F, E y D es 60º, además, Solución se forman los triángulos AFM y DEN; estos, a la Tema vez, forman el triángulo isósceles MBCN, donde Cono circular recto MB=CN. Como Referencias DE=8 → DN=EN=8. Al desarrollar la superficie lateral de un cono Así también si circular recto, resulta un sector circular cuyos elementos se asocian con los del cono dado. AF=a → AM=MF=a. A Luego g a+7=6+8 V ∴ a=7 a Por lo tanto, en el triángulo notable BAF tenemos 2pr B g g h A r 7 120º 7 B O A F x B En el gráfico α es la medida del ángulo de Entonces, BF=7 3. desa-rrollo. Sea θ su medida en radianes. Respuesta πα → θ= 180º La longitud de BF es 7 3. Luego, la longitud del arco ABA se asocia con el Alternativa E radio de la base del cono. 26
  • 13. Matemática ´ ABA =2πr Solución Tema ´ ABA =θ×g Sistemas de medición angular 2πr ∴ θ= g Referencias Análisis y procedimiento La equivalencia entre los grados sexagesimales y el número de radianes de un ángulo es π rad=180º. Nos dan como dato α=120º y h=4 cm; entonces, podemos calcular θ y encontrar una relación entre Análisis y procedimiento r y g. • Nuevo sistema de medición angular (X), donde 1X denota un grado en el sistema X. π (120º ) 2π → θ= = 180º 3 • Condiciones: Luego αº=(α – 3)X π rad=120X r 1 ó g=3r = Empleamos el método del factor de conversión: g 3 Como nos piden el radio de la base en cm, re- ⎛ π rad ⎞ ⎛ 180º ⎞ αº = (α − 3) X ⎜ ⎟⎜ ⎟ ⎝ 120 X ⎠ ⎝ π rad ⎠ currimos al teorema de Pitágoras para relacionar r, g y h. º ⎛3⎞ En el AVO: g 2=r 2+h2 αº = (α − 3) ⎜ ⎟ ⎝ 2⎠ Reemplazamos valores: 2α=3α – 9 (3r)2=r 2+(4)2 α=9 ∴ r= 2 Se busca calcular (α – 3). Respuesta El radio del cono en centímetros es 2. Respuesta Alternativa B El valor de (α – 3) es 6. Alternativa B Pregunta N.º 33 En un nuevo sistema de medición angular, un ángulo de α grados sexagesimales mide α – 3. Si Pregunta N.º 34 a 3 un ángulo de π radianes mide 120 en el nuevo En la figura = y el área de la región sombreada b 2 sistema, halle α – 3. es 5 veces el área del sector circular OPQ. ´ SR Determine la relación . A) 3 B) 6 C) 9 ´ BA D) 12 E) 15 27
  • 14. Matemática 2k D B 3k Q S O q a P A R 2 16 3 C A) B) C) 3 27 2 Pero ´ SR = α(5k) 45 10 D) E) 16 3 ´ BA = θ(3k) Solución ´ SR 5 ⎛ α ⎞ Tema = (I) ´ BA 3 ⎜ θ ⎟ ⎝ ⎠ Longitud de arco y área del sector circular Condición 2 Referencias El área sombreada es igual a cinco veces el área • Longitud de arco (´) del sector OPQ. 1 1 ⎛ α(3k)2 ⎞ r θ(5k)2 − θ(3k)2 = 5 ⎜ 2 2 ⎜ 2 ⎟ ⎟ ⎝ ⎠ q rad µ µ=q×r 16θk 2 45αk 2 = 2 2 • Área de un sector circular (A) 16 α = (II) 45 θ r Al reemplazar (II) en (I) se obtiene: q rad A=qr 2 2 ´ SR 5 ⎛ 16 ⎞ = ´ BA 3 ⎜ 45 ⎟ ⎝ ⎠ ´ SR 16 Análisis y procedimiento = ´ BA 27 Condición 1 a 3 a = 3k Respuesta = b 2 b = 2k ´ SR 16 La relación es . ´ BA 27 ´ SR Incógnita: ´ BA Alternativa B 28
  • 15. Matemática Pregunta N.º 35 Reemplazamos (II) en (I) Un punto M=(x; y) dista de un punto C=(2; 5), 2 ⎛1 ⎞ (x – 2)2+⎜ ( x − 7 ) ⎟ = 10 10 unidades. La pendiente de la recta que pasa ⎝2 ⎠ por M y A=(7; 5) es 1/2. Determine el punto M 1 de mayor abscisa. (x – 2)2+ (x – 7)2=10 4 Reduciendo, tenemos A) (–1; 4) B) (–1; 6) C) (1; 8) D) (3; 2) E) (5; 4) x2 – 6x+5=0 x –5 Solución x –1 Tema x=5 ∨ x=1 Piden el punto M de mayor abscisa< enton- Geometría analítica ces, x=5. Referencias Reemplazamos en (II) 1 • Distancia entre dos puntos y – 5= (5 – 7) 2 • Ecuación de una recta y=4 Entonces, M=(5,4). Análisis y procedimiento De la condición tenemos Respuesta • C(2; 5) El punto M de mayor abscisa es (5,4). 10 Alternativa E M (x ; y ) Por distancia entre dos puntos se cumple que Pregunta N.º 36 2 2 En el círculo trigonométrico de la figura, se tiene 10 = ( x − 2 ) + ( y − 5 ) CM = DM . Entonces el área de la región triangular Elevando al cuadrado, tenemos ABM es: (x – 2)2+(y – 5)2=10 (I) 1 • Dato m = L 2 L A(7; 5) M Calculamos la ecuación de la recta L . y – 5=m (x – 7) L 1 y – 5= (x – 7) (II) 2 29
  • 16. Matemática ⎛ 3π ⎞ En el gráfico se observa que AB= 2 y AM=BM, A) 2 tan ⎜ ⎟ ⎝ 8 ⎠ 2 entonces, AH=HB= . 1 ⎛ 3π ⎞ 2 B) tan ⎜ ⎟ 2 ⎝ 8 ⎠ Calculamos la altura MH en el triángulo AHM. ⎛ 3π ⎞ 2 3π C) 2 tan ⎜ ⎟ ⎝ 4 ⎠ MH = tan 2 8 1 ⎛ 3π ⎞ Luego D) tan ⎜ ⎟ 2 ⎝ 4 ⎠ ( AB)(MH ) S= ⎛ 4π ⎞ 2 E) 2 tan ⎜ ⎟ ⎝ 7 ⎠ ⎛ 2 3π ⎞ ( 2)⎜ ⎜ 2 tan ⎟ 8 ⎟ S= ⎝ ⎠ Solución 2 Tema Por lo tanto, Circunferencia trigonométrica (C. T.) 1 3π S= tan . Referencias 2 8 • Ubicación de arcos en la C. T. Respuesta • Resolución de triángulos rectángulos. • Cálculo del área de una región triangular. El área de la región triangular ABM es igual a 1 3π tan . Análisis y procedimiento 2 8 π Alternativa B Dato: CM = DM → mCM = m DM = 4 π π 3π además, m BM = + → m BM = . 2 4 4 Pregunta N.º 37 Simplificando la siguiente expresión Y K=sen23Acsc2A+cos23Asec2A+2cos4A, 3p C se obtiene 4 M A) 6cos22A B) 6cos2A B C) 8sen2A D X D) 12senA 2 E) 12cos22A 3p 2 H 8 2 Solución A 2 Tema Identidades trigonométricas de arcos múltiples 30
  • 17. Matemática Referencias Solución • Empleamos las identidades auxiliares del arco Tema triple Funciones trigonométricas sen3θ=senθ(2cos2θ+1) cos3θ=cosθ(2cos2θ – 1) Referencias • Empleamos la identidad del arco doble relacio- Para reducir la expresión aplicaremos identidades nada con el coseno. trigonométricas. cos2θ=2cos2θ – 1 sen x cos x tan x = cot x = cos x sen x Análisis y procedimiento K=sen23Acsc2A+cos23Asec2A+2cos4A Análisis y procedimiento sen x + tan x π entonces f ( x) = x≠K cos x + cot x 2 2 2 ⎛ sen 3 A ⎞ ⎛ cos 3 A ⎞ K =⎜ ⎟ +⎜ ⎟ + 2 cos 4 A ⎝ sen A ⎠ ⎝ cos A ⎠ cosx+cotx ≠ 0 Ahora aplicamos las identidades del arco triple. cosx(1+1/senx) ≠ 0 K=(2cos2A+1)2+(2cos2A – 1)2+2cos4A cosx ≠ 0 ∧ senx ≠ – 1 Desarrollando los binomios y aplicando la identi- π → x ≠ (2n+1) dad del arco doble, obtenemos 2 K=2(4cos22A+1)+2(2cos22A – 1) sen x → K=12cos22A sen x + f ( x) = cos x cos x cos x + Respuesta sen x Entonces, K es igual a 12cos22A. ⎛ 1 + cos x ⎞ sen x ⎜ ⎟ ⎝ cos x ⎠ f ( x) = ⎛ 1 + sen x ⎞ cos x ⎜ ⎟ Alternativa E ⎝ sen x ⎠ sen 2 x (1 + cos x ) f (x) = cos 2 x (1 + sen x ) Pregunta N.º 38 senx > – 1 → 1+senx > 0 sen x + tan x π cosx > – 1 Sea f ( x ) = , x≠k . cos x + cot x 2 → 1+cosx > 0 Entonces podemos afirmar que Entonces, se deduce que f(x) es positivo. A) f(x) toma valores positivos y negativos. Respuesta B) f(x) toma un número finito de valores negativos. f(x) toma solamente valores positivos. C) f(x) toma solamente valores negativos. D) f(x) toma solamente valores positivos. Alternativa D E) f(x) es constante. 31
  • 18. Matemática Pregunta N.º 39 ⎛ x−y⎞ ⎛ x−y⎞ → 4·cos 2 ⎜ ⎟ + 4 cos ⎜ ⎟−3=0 Dado el sistema ⎝ 2 ⎠ ⎝ 2 ⎠ ⎧ 4π ⎛ ⎛ x−y⎞ ⎞⎛ ⎛ x−y⎞ ⎞ ⎪ x+y= ⎜ 2 cos ⎜ ⎟ + 3 ⎟ ·⎜ 2 cos ⎜ ⎟ − 1⎟ = 0 ⎨ 3 ⎝ ⎝ 2 ⎠ ⎠⎝ ⎝ 2 ⎠ ⎠ ⎪sec x + sec y = 1 ⎩ ⎛ x−y⎞ 1 cos ⎜ ⎟= o el valor de cos(x – y) es: ⎝ 2 ⎠ 2 ⎛ x−y⎞ 3 1 1 1 cos ⎜ ⎟=− A) − B) − C) − ⎝ 2 ⎠ 2 4 3 2 La ecuación admite para 1 1 D) E) 4 2 ⎛ x−y⎞ 1 cos ⎜ ⎟= ⎝ 2 ⎠ 2 Solución Tema Luego, debido a que Sistemas de ecuaciones trigonométricas ⎛ x−y⎞ cos ( x − y ) = 2 cos 2 ⎜ ⎟ −1 ⎝ 2 ⎠ Referencias Por lo tanto Transformaciones trigonométricas. 1 ⎛ x+y⎞ ⎛ x−y⎞ cos ( x − y ) = − cos x + cos y = 2 cos ⎜ ⎟ ·cos ⎜ ⎟ 2 ⎝ 2 ⎠ ⎝ 2 ⎠ Respuesta Identidad de arco doble. 1 cos2x=2cos2x – 1 El valor de cos(x – y) es − . 2 Análisis y procedimiento Alternativa C De la condición secx+secy=1 Pregunta N.º 40 2 · (cosx+cosy)=2(cosx · cosy) En las circunferencias tangentes de la figura, son datos r0 (radio) y α. Determine el radio R. ⎛ x +y⎞ ⎛ x −y⎞ 2× 2 ·cos ⎜ ⎟ ·cos ⎜ ⎟ = cos ( x + y ) + cos ( x − y ) ⎝ 2 ⎠ ⎝ 2 ⎠ Por dato sabemos que 4π x+y= . 3 ⎛ −1 ⎞ ⎛ x−y⎞ 1 2⎛ x −y⎞ 4 ⎜ ⎟ cos ⎜ ⎟ = − + 2 cos ⎜ ⎟ −1 ⎝ 2 ⎠ ⎝ 2 ⎠ 2 ⎝ 2 ⎠ 32
  • 19. Matemática ⎛ 1 − cos α ⎞ Análisis y procedimiento A) ⎜ ⎟ r0 ⎝ cos α ⎠ ⎛ cos α ⎞ r0 B) ⎜ ⎟ r0 ⎝ 1 − cos α ⎠ R ⎛ 1 − cos α ⎞ C) ⎜ ⎟ r0 ⎝ 1 + cos α ⎠ a R ⎛ 1 + cos α ⎞ Por definición tenemos D) ⎜ ⎟ r0 R ⎝ cos α ⎠ cos α = R + r0 Rcosα+r0cosα=R ⎛ 1 + cos α ⎞ E) ⎜ ⎟ r0 ⎝ 1 − cos α ⎠ r0cosα=R(1 – cosα) r cos α R= 0 1 − cos α Solución ⎛ cos α ⎞ Tema R=⎜ ⎟ r0 ⎝ 1 − cos α ⎠ Razones trigonométricas de un ángulo agudo Respuesta Referencias Entonces, el radio R, en términos de r0 y α, es Definición del coseno de un ángulo agudo. ⎛ cos α ⎞ ⎜ ⎟ r0 cateto adyacente ⎝ 1 − cos α ⎠ cos α = hipotenusa Alternativa B 33